Sie sind auf Seite 1von 87

No.

1 for CA/CWA & MEC/CEC

MASTER MINDS

PART 1. INTRODUCTION TO SERVICE TAX


NO. 1. 2. 3. 4. 5. 6. 7. 8. 9. 10. 11. QUESTION What is the need for the introduction of Service Tax? Evaluation of Service Tax in India. What is the constitutional back ground of India? How is the Service Tax law administered in India? Explain the applicability of geographical areas of Service Tax. Explain the features of Service Tax. Explain briefly the incidence for levy and collection of Service Tax. Exceptions to the rule Service Tax is to be paid within 14 days from invoice issued or the receipt of payment whichever is earlier. Write short notes on classification of services. What are the statutes governing Service Tax? Special Economic Zone. ABC C C B B A A A A C B C

Q.NO.1. WHAT IS THE NEED FOR THE INTRODUCTION OF SERVICE TAX? It is the prime responsibility of the Government to fulfill the increasing development needs of the country and its people, by way of public expenditure. The Governments primary sources of revenue are direct and indirect taxes. Central Excise Duty on the goods manufactured and produced in India and Customs Duties on imported goods constitute the two major sources of indirect taxes in India. Due to WTO commitments and rationalization of commodity duties, the revenue receipts from customs and excise duties are low. Q.NO.2. EVALUATION OF SERVICE TAX IN INDIA. Dr.Manmohan Singh, the then Union Finance Minister, in his Budget Speech for the year 1994-95, introduced the new concept of Service Tax and stated as under: There is no sound reason for exempting services from taxation, where goods are taxed and many countries treat goods and services alike for tax purposes. I, therefore, propose to make a modest effort in this direction by imposing a tax on services of TELEPHONE, NON-LIFE INSURANCE, AND STOCK BROKERS. Therefore, the Service Tax was levied under Chapter V of the Finance Act, 1994. It was introduced for the first time on 3 services with a nominal rate of 5% advalorem basis. Subsequent Finance Acts have added more and more services to be taxed for Service Tax purposes. As such, today, more than 100 services are chargeable to Service Tax.

IPCC _27e_Service Tax_Finance Act 2011 ____________________________ 7

Ph:

0863 22 42 355

www.gntmasterminds.com

Q.NO.3. WHAT IS THE CONSTITUTIONAL BACKGROUND OF INDIA? According to Article 265 of the constitution India, no tax of any nature can be levied or collected by Central or State Governments, except by the Authority of Law. According to Article 246, law can be enacted by the Parliament or the State Legislature, if such power is given by the Constitution of India. List I List II List III Union list Parliament has the exclusive right to make law in respect of that entry. State list Any state has exclusive power to make law for such state or any part there of with respect to such an entry. Concurrent list The parliament or the legislature of a state has power to make laws with respect to any matter, enumerated in List III.

There are various matters enumerated in each list. Each matter in the list is known as an entry. Entry 97 of the Union list is the residuary entry and it empowers the Central Government to levy tax on any matters, which are not enumerated in List II (State List) or List III (Concurrent List). In 1994, the Service Tax was levied by the Central Government, under the powers granted under the said Entry 97, of List I. Entry 92C has been inserted to the 1st List, in the VII Schedule so as to make the enactment a subject matter of Union List. Although the Government has amended the Constitution and inserted entry No.92C in the List 1 of Schedule VII no separate Act has been passed yet and Service Tax is still being governed by entry 97 i.e. residuary entry. Q.NO.4. HOW IS THE SERVICE TAX LAW ADMINISTERED IN INDIA? 1. Controlling Authority: The responsibility of administration and collection of Service Tax has also been vested upon the Central Board of Excise and Customs. 2. Administering Authority: The Board administers Service Tax matters, through the Central Excise Zone. Each Zone, in turn works through Central Excise Commissionerate, falling under its territory. 3. Zonal Head: Each zone is headed by a Chief Commissioner of Central Excise, while each Commissionerate is headed by a Commissioner of Central Excise. 4. Role of Zonal Head: The Chief Commissioner of Zone exercises supervision and control over the working of the Commissionerates in the Zone and is mainly responsible for monitoring revenue collection, disposal of pendencies, redressal of grievances of trade, etc. He also ensures coordination among the Commissionerates, within the Zone.

IPCC _27e_Service Tax_Finance Act 2011 ____________________________ 8

No.1 for CA/CWA & MEC/CEC

MASTER MINDS

PICTORAL OVERVIEW OF SERVICE TAX ADMINISTRATION Ministry of Finance (1) Department of Revenue (2) Central Board of Excise & Customs (3) Central Excise Zones headed by chief Commissioners (4) Central Excise Commissionerates headed by Commissioners (5)

Director General of Service Tax (Coordinator between 3 & 5)

Q.NO.5. EXPLAIN THE APPLICABILITY OF GEOGRAPHICAL AREAS OF SERVICE TAX. 1. Destination Based: Service Tax is destination based and service is taxable only if provided in India. Hence, in the following cases, there is no Service Tax liabilitya. Technical Consultancy provided by Foreign Collaborator (Illustrative). Such service is not provided in India. However, if the foreign technicians visit India and provide technical services, tax will be payable (subject to import of service rules -2005). b. If Indian service provider provides services abroad. c. Services consumed for the purpose of Export. 2. Special Consideration for Jammu & Kashmir: The levy of Service Tax extends to the whole of India except Jammu & Kashmir. The following are the observations in this regarda. Services rendered in Jammu and Kashmir will not be liable to Service Tax. b. Service rendered by a person established in Jammu and Kashmir, but rendered outside the state is liable to Service Tax. 3. Service provided in Exclusive Economic Zone and Continental shelf: India includes territorial waters and exclusive economic zone. Therefore Service Tax provisions are applicable to designated areas of continental shelf and exclusive economic zone of India, which extends upto 200 nautical miles inside the sea from the base line. 4. Service rendered in Indian territorial waters upto 12 nautical miles from the Indian land mass is taxable. Q.NO.6. EXPLAIN THE FEATURES OF SERVICE TAX. The salient features for the levy of Service Tax are: 1. Scope: It is leviable on taxable services 'provided' or 'to be provided', by a service provider. The services 'to be provided' in future are also taxed Two separate persons required - Payment to employees not covered: For charging service tax, it is necessary that the service provider and service recipient should be two separate persons, acting on 'principal-to-principal basis'. Services, provided by an employee to his employer, are not covered under Service Tax. Therefore, salaries or allowances paid to the employees are not charged to Service Tax.

IPCC _27e_Service Tax_Finance Act 2011 ____________________________ 9

Ph:

0863 22 42 355

www.gntmasterminds.com

2. Rate: It is leviable @ 10%, of the value of taxable services. Education Cess @ 2% and Secondary and Higher Education Cess @ 1 % are chargeable on the amount of Service Tax. Therefore, effective rate of Service Tax is 10.3% of the value of taxable service. 3. Free services not taxable: Service Tax is not levied upon the services provided free of cost. 4. Services provided by an unincorporated association/body to its members are also taxable [Explanation to Sec. 65]: 'Taxable service' includes any taxable service provided or to be provided, by any unincorporated association or body of persons, to a member thereof, for cash, deferred payment or any other valuable consideration. Hence, the services (falling under any category of taxable service) provided or to be provided by any unincorporated association / body, to a member thereof, shall be liable to Service Tax. This provision is an exception to the 'principle of mutuality'. 5. Performance of statutory activities/duties, not 'service': An activity performed by a sovereign / public authority, under the provisions of law does not constitute provision of taxable service, to a person. Therefore, Service Tax is not levied on such entities. Q.NO.7. EXPLAIN BRIEFLY THE INCIDENCE FOR LEVY AND COLLECTION OF SERVICE TAX. Finance Act, 2009 has amended Section 94 in order to empower the Central Government to frame Rules for the date for determination of a rate of Service Tax and the place of provision of taxable service. The Central Government has framed the point of Taxation Rules, 2011 which specifically deals with changes in rate of tax. Point of Taxation: Earlier the payment of Service Tax was only when there was receipt of Payments. Now, w.e.f 1st April, 2011 or 1st July, 2011 as the case may be, a. If invoice is issued within 14 days of the completion of service then the point of Taxation shall be the receipt of Payment or Issue of Bill, whichever is earlier. b. If no invoice is issued within 14 days of the completion of service then the Point of Taxation shall be deemed to be the date of completion of such service (Main + Auxiliary activities) Actual Position with reference to Service / Provision of Service Taxable service has been provided before the change in effective rate of tax. Taxable service has been provided before the change in effective rate of tax. Invoice / Payment Status Invoice has been issued and payment received after the change in effective rate of tax Invoice has been issued before the change in effective rate but payment is received after the change in effective rate. Payment has been received before the change in effective rate but invoice has been issued after the change in effective rate. Effective Rate and Point of Taxation Point of Taxation shall be the date of payment or issue of invoice whichever is earlier. Point of Taxation shall be date of issuing of invoice.

Taxable service has been provided before the change in effective rate of tax.

Point of Taxation shall be the date of payment.

IPCC _27e_Service Tax_Finance Act 2011 ____________________________ 10

No.1 for CA/CWA & MEC/CEC


Taxable service has been provided after the change in effective rate of tax. Payment is made after the change in effective rate but invoice has been issued before the change in effective rate. Invoice has been issued and payment received before the change in effective rate. Invoice has been issued after the change in effective rate but payment is received before the change in effective rate.

MASTER MINDS
Point or Taxation shall be the date of payment.

Taxable service has been provided after the change in effective rate of tax. Taxable service has been provided after the change in effective rate of tax.

Point of Taxation shall be the receipt of payment or date of issue of invoice whichever is earlier. Point of Taxation shall be the date of issue of invoice.

Effective rate of tax shall include a change in the portion of value on which tax is payable in terms of a Notification issued under the Act or Rules. To illustrate, assuming an abatement of 67% is granted and the same is changed to 70%, that change in value would also be considered as an effective rate change for the purpose of application of point of Taxation Rules. Taxable, only when there are two parties: Service Tax is attracted when there are two parties. One cannot give service to himself. Tax Liability will be there, even if the Service Tax is not paid by the service recipient. Rolls Royce Indus Power (2004) 171 ELT 189 (CESTAT) Wild Expeditions Tours and Travels (2006) 4 STR 58 (CESTAT)

Q.NO.8. EXCEPTIONS TO THE RULE SERVICE TAX IS TO BE PAID WITHIN 14 DAYS FROM INVOICE ISSUED OR THE RECEIPT OF PAYMENT WHICHEVER IS EARLIER.

POINT OF TAXATION IN CASE OF SPECIFIED SERVICES OR PERSONS


1. Individuals / Proprietary firms / Partnership firms: Point of Taxation shall be on receipt of payment in case of individuals, proprietary Firms and Partnership Firms for providing below mentioned services: Consulting Engineers Services [Section 65(105)(g)] Architects Services [Section 65(105)(p)] Interior Decorators Services [Section 65(105)(q)] Practicing Chartered Accountants Services [Section 65(105)(s)] Practicing Cost Accountants Services [Section 65(105)(t)] Practicing Company Secretarys Services [Section 65(105)(u)] Scientific or Technical Consultancy Services [Section 65(105)(za)] Legal Consultancy Services [Section 65(105)(zzzzm)] 2. Export of Services: Point of Taxation shall be on receipt of payment for services exported as per Export of service rules, 2005. However, if the payment for exports is not received within the period specified by the RBI (Currently it is 12 months), then the point of Taxation shall be as applicable as per general rule.

IPCC _27e_Service Tax_Finance Act 2011 ____________________________ 11

Ph:

0863 22 42 355

www.gntmasterminds.com

3. Receiver of Services U/s. 68(2): Point of Taxation shall be the date of payment. However, if the payment is not made within 6 months of the date of invoice, then the point of Taxation shall be as applicable as per general rule. 4. Associated Enterprises: Service provider is located outside India: Point of Taxation shall be date or credit in the books of account of the service receiver or date of payment, whichever is earlier. Q.NO.9. WRITE SHORT NOTES ON CLASSIFICATION OF SERVICES. 1. Classification: Classification of taxable services shall be determined according to the terms of Section 65(105) i.e., by the definition of various services. 2. More than one classification: Where the service falls under more than one category of service, a. Prefer specific description to general description. b. In the case of composite service, go by the service, which gives the essential character. c. Where classification is not possible under the above two rules, and the service falls under more than one category U/s 65(105), then it shall be classified under the subclause, which occurs first among the sub-clauses, with equal merit consideration. (Sec. 65A of the Act) Q.NO.10. WHAT ARE THE STATUTES GOVERNING SERVICE TAX? Laws relating to Service Tax: An understanding of the Service Tax law requires the study of the following: 1. Chapter V and V A, of the Finance Act, 1994, amended upto date. 2. Rules on Service Tax. 3. Notifications, issued by the Central Government, from time to time. 4. Circulars and clarifications, issued by Central Board of Excise and Customs (CBEC). 5. Trade Notice, issued by respective Jurisdictional Commissionerate. 6. Definitions given under other statutes. 7. Judicial decisions. Q.NO.11. SPECIAL ECONOMIC ZONE (SEZ)? Special Economic Zone: All earlier Notifications have been withdrawn and the Government of India has introduced Notification No.17/2011 w.e.f. 01.03.2011. The exemption shall be provided by way of refund of Service Tax paid on the specified service received for the authorized operations in a SEZ. Where the specified services received and used for authorized operations are wholly consumed within the SEZ, the provider of such services or the receiver of such services under reverse charge basis as the case may be has the option not to pay Service Tax abinitio instead of a unit or a developer claiming exemption by way of refund in terms of this Notification.

IPCC _27e_Service Tax_Finance Act 2011 ____________________________ 12

No.1 for CA/CWA & MEC/CEC Illustrations

MASTER MINDS

1. Computation of Service Tax: A service provider provides taxable services valuing Rs. 10,00,000. Compute the amount of Service Tax payable by him. Particulars Service Tax payable @ 10% Add: Education Cess @ 2% of Service Tax Add: SH & EC @ 1% of Service Tax Total Service Tax Payable Amount Rs.1,00,000 Rs.2,000 Rs.1,000 Rs.1,03,000

2. Computation of Service Tax: A service provider provides taxable services. The gross amount charged, is Rs. 10,00,000. Compute the amount of Service Tax payable by him, if the gross amount so charged in inclusive of Service Tax. Solution: Service Tax payable = 10,00,000 10.3 110.3 = Rs. 93,382. 3. Computation of Service tax: A service provider provided taxable services on 1-7-2011, the gross amount charged in respect of which is Rs.10,00,000(Excluding service Tax). The bill was raised on 13-07-2011 and payment was received on 15-07-2011. Compute the amount of Service Tax payable by him and when?(Assumed to be a Company) Solution: Particulars Gross Amount charged Service Tax payable @ 10% Add: Education Cess @ 2% of Service Tax Add: SH & EC @ 1% of Service Tax Total Service Tax Payable Amount 10,00,000 1,00,000 2,000 1,000 1,03,000

Effective Date = If invoice is issued within 14 days of the completion of service then the point of Taxation shall be the receipt of Payment or Issue of Bill, whichever is earlier. Date of completion = 1-07-2011 and Invoice date = 13-07-2011.Hence, Invoice is issued within 14days of completion. So, Effective Date = 13-7-11 or 15-7-11 whichever is earlier = 13-07-2011 Due date = on or before 05-08-2011 4. A particular service has been brought into the Service Tax net, with effect from 16.05.2011. Mr.Vignesh has provided this service on 20.04.2011 and Invoice Date is 04-05-2011.Payment for the same was received on 10.06.2011. Is Service Tax payable on the same? Solution: 1. Effective Date = If invoice is issued within 14 days of the completion of service then the point of Taxation shall be the receipt of Payment or Issue of Bill, whichever is earlier. 2. The service is not taxable, even if the bills are raised or payment received, after the effective date. 3. Conclusion: Therefore, Service Tax is not payable on the service provided by Mr.Vignesh on 20.04.2011.

IPCC _27e_Service Tax_Finance Act 2011 ____________________________ 13

Ph:

0863 22 42 355
Self Test Questions

www.gntmasterminds.com

1. Find out the amount of Service Tax in the following cases


Case 1 Service provider is X Ltd. which is based in Jammu and Kashmir. However,

the services are provided in the state of Karnataka (amount of invoice being Rs. 18,50,000)
Case 2 Service is provided by Y Ltd. to UNO in Delhi (amount of invoice being Rs.

2,00,000)
Case 3 Service is provided by Z Ltd. to a unit in a Special Economic Zone (amount of

invoice being Rs. 2,00,000)


Case 4 Service is provided by A Ltd. was incorporated in 2011. Since then its annual

turnover/gross receipts is not more than Rs. 6,00,000 (amount of invoice being Rs. 2,50,000) 2. Under which Act, the Service Tax is levied? 3. At what rate the Service Tax is levied? 4. When is the Service Tax payable? 5. Is Service Tax payable on advances received by service provider? 6. Are services provided in the State of Jammu and Kashmir liable for Service Tax? 7. Is Service Tax payable on services provided to a developer or an unit in the Special Economic Zone (SEZ)? 8. State briefly whether the following services under the Finance Act, 1994 relating to Service Tax are taxable services? a. Services provided in the State of Rajasthan, by a person having a place of business in the State of Jammu and Kashmir. b. Services provided from India, for use outside India. c. Service provided from outside India and received in India by an individual, otherwise than for purpose of use in business or commerce. d. Service provided to an Export Oriented Unit.

IPCC _27e_Service Tax_Finance Act 2011 ____________________________ 14

No.1 for CA/CWA & MEC/CEC

MASTER MINDS

PART 2. TAXABLE SERVICES


No. 1. 2. 3. Question Private tuition centers are within the scope of levy of Service Tax. Explain Members of the commerce profession i.e., chartered accountants, company secretaries, cost accountants are liable to pay Service Tax for services rendered by them discuss. [sec 65 (105) (u), 65 (105) (t)] Services rendered by professional engineers are liable to Service Tax. Explain. [sec 65(105)(g)] Chartered Accountant plays a significant role in the matters involving Service Tax explain Information Technology Services are subject to Service Tax. Explain Who are liable to pay service tax under Scientific or Technical Consultancy Service? [Section 65(105)(za)] Examine the applicability of service tax law to Technical Testing And Analysis Service. [Sec 65(105)(zzh)] Examine the applicability of service tax law to Business Exhibition Services Examine the applicability of service tax law to Mandap Keepers Services ABC A A

A B B A A A A

4. 5. 6. 7. 8. 9.

No. 1. 2. 3. 4. 5. 6. 7. 8. 9. 10. 11. 12. 13. 14. 15. 16.

Sec. No 65 (105) (e) 65 (105)(zzzo) 65 (105) (l) 65 (105) (zzm) 65 (105) (p) 65 (105) (zzzc) 65 (105) (zzzr) 65 (105) (zo) 65 (105) (zzzk) 65 (105) (zm) 65 (105) (zq) 65 (105) (zk) 65 (105) (zzb) 65 (105) (zzo) 65 (105) (zzzq) 65 (105) (zs)

Taxable Services Advertising Agencys Services Air Transport of Passengers embarking for International travel Air Travel Agents Services Airport Services Architects Services Asset Management Services Auctioneers Services Authorised Service Stations Services Automated Teller Machine (ATM) Operations, Maintenance or Management Services Banking and other Financial Services Beauty Treatment Services Broadcasting Agency Services Business Auxiliary Services Business Exhibition Services Business support services Cable Services

Taxable w.e.f 01.11.1996 01.05.2006 01.07.1997 10.09.2004 16.10.1998 12.05.2007 01.05.2006 16.07.2001 01.05.2006 16.07.2001 16.08.2002 16.07.2001 01.07.2003 19.09.2004 01.05.2006 16.08.2002

IPCC _27e_Service Tax_Finance Act 2011 ____________________________ 15

Ph:

0863 22 42 355
65 (105) (zr) 65 (105) (s) 65 (105) (zzzd) 65 (105) (j) 65 (105) (zzc) 65 (105) (u) 65 (105) (zzq) 65 (105) (zzzh) 65 (105) (g) 65 (105) (zc) 65 (105) (t) 65 (105) (zzzzk) 65 (105) (f) 65 (105) (zzzw) 65 (105) (x) 65 (105) (h) 65 (105) (zzzzd) 65 (105) (zzzzb) 65 (105) (zzzb) 65 (105) (zt) 65 (105) (zzd) 65 (105) (zu) 65 (105) (zv) 65 (105) (zzy) 65 (105) (zze) 64 (105) (d) 65 (105) (zzp) 65 (105) (zw) 65 (105) (zzzze) 65 (105) (zy) 65 (105) (zl) 65 (105) (zzr) 65 (105) (q)

www.gntmasterminds.com
16.08.2002 16.10.1998 16.06.2005 16.07.1997 01.07.2003 16.10.1998 10.09.2004 16.06.2005 07.07.1997 16.07.2001 16.10.1998 01.09.2009 01.11.1996 01.05.1996 16.07.1997 15.06.1997 12.05.2007 12.05.2007 16.06.2005 16.08.2002 01.07.2003 16.08.2002 16.08.2002 10.09.2004 01.07.2003 01.07.2004 01.01.2005 16.08.2002 16.05.2008 16.07.2001 16.07.2001 10.09.2004 16.10.1998

17. 18. 19. 20. 21. 22. 23. 24. 25. 26. 27. 28. 29. 30. 31. 32. 33. 34. 35. 36. 37. 38. 39. 40. 41. 42. 43. 44. 45. 46. 47. 48. 49.

Cargo Handling Agencys Service Chartered Accountants Services Cleaning services Clearing and Forwarding Agents Services Commercial Training or Coaching Centres Services Company Secretarys Services Construction Services Commercial or industrial Construction services-Residential Complex Consulting Engineers Services Convention Services Cost Accountants Services Cosmetic or Plastic Surgery Services Courier Agencys Services Credit Card, Debit Card, Charge Card or other Payment Card Services Credit Rating Agencys Services Custom house agents Services Design Services Development and Supply of Content Services Dredging Services Dry Cleaning Services Erection, Commissioning or Installation Agencys Services Event Management Services Fashion Designing Services Forward Contract Services Franchise Services General Insurance Business Services Goods Transport Agencys service Health & Fitness Services Information Technology Services Insurance Auxiliary Services (Concerning General Insurance) Insurance Auxiliary Services ( Concerning Life Insurance) Intellectual Property Services Interior Decorators services

IPCC _27e_Service Tax_Finance Act 2011 ____________________________ 16

No.1 for CA/CWA & MEC/CEC


50. 51. 52. 53. 54. 55. 56. 57. 58. 59. 60. 61. 62. 63. 64. 65. 66. 67. 68. 69. 70. 71. 72. 73. 74. 75. 76. 77. 78. 79. 80. 81. 82. 83. 84. 65 (105) (zzf) 65 (105) (zzzu) 65 (105) (zzzu) 65 (105) (zx) 65 (105) zzzzm) 65 (105) (zzzg) 65 (105) (r) 65 (105) (zzg) 65 (105) (m) 65 (105) (k) 65 (105) (y) 65 (105) (zzze) 65 (105) (zzzy) 65 (105) (zh) 65 (105) (zzs) 65 (105) (zzl) 65 (105) (zzt) 65 (105) (zzzf) 65 (105) (zzw) 65 (105) (zb) 65 (105) (zn) 65 (105) (zzzzi) 65 (105) (zzu) 65 (105) (zzzs) 65 (105) (zz) 65 (105) (v) 65 (105) (zzzzh) 65 (105) (zzzzg) 65 (105) (zzzl) 65 (105) (zzzi) 65 (105) (o) 65 (105) (zzzz) 65 (105) (zzzm) 65 (105) (za) 65 (105) (w) Internet Caf Services Internet Telecommunication Services Internet Telephony Services Life Insurance Services Legal Consultancy and Advice Service

MASTER MINDS
01.07.2003 16.05.2008 01.05.2006 16.08.2002 01.09.2009 16.06.2005 16.07.1997 01.07.2003 01.07.1997 07.07.1997 16.10.1998 16.06.2005 12.05.2007 16.07.2001 10.09.2004 16.07.2001 10.09.2004 16.06.2005 10.09.2004 16.07.2001 16.07.2001 16.05.2008 10.09.2004 16.07.1997 16.08.2002 16.10.1998 16.05.2008 16.05.2008 01.05.2006 01.05.2006 16.07.1997 12.05.2007 01.05.2006 16.07.2001 16.10.1998

Mailing List Compilation and Mailing services Management or Business Consultants Services Management, Maintenance or Repair Services Mandap Keepers services Manpower Recruitment or Supply Agencys Services Market Research Agencys Services Membership of Clubs or Associations Services Mining of mineral, oil or gas services On-line Information and Database Access or Retrieval Services Opinion Poll Agencys Services Other Port Services Outdoor Catering Services Packaging Services Pandal or Shamiana Services Photography Services Port Services Processing and Clearing House Services Programme Producers Services Public Relation Services Rail Travel Agents Services Real Estate Agents Services Recognised Association Services Recognised stock Exchange Services Recovery Agents Services Registrar to an Issue Services Renting of Cab Services Renting of Immovable Property Services Sale of Space or Time for Advertisement Services Scientific or Technical Consultancy Services Security Agencys Services

IPCC _27e_Service Tax_Finance Act 2011 ____________________________ 17

Ph:

0863 22 42 355
65 (105) (zzzj) 65 (105) (zzzt) 65 (105) (zzza) 65 (105) (zj) 65 (105) (zzzn) 65 (105) (i) 65 (105) (a) 65 (105) (zza) 65 (105) (zzzzj) 65 (105) (zzv) 65 (105) (zzzc) 65 (105) (zzi) 65 (105) (zzh) 65 (105) (zzzx) 65 (105) (n) 65 (105) (zzn) 65 (105) (zzz) 65 (105) (zzzp) 65 (105) (zzzv) 65 (105) (zzzzl) 65 (105) (zzx) 65 (105) (zzzzf) 65 (105) (z) 65 (105) (zi) 65 (105) (zzzza) 65 (105) zzzzr) 65 (105) (zzzzp) 65 (105) (zzzzo) 65 (105) (zzzzu) 65 (105) (zzzzn) 65 (105) (zzzzq) 65 (105) (zzzzt) 65 (105) (zzzzs) Ship Management Services

www.gntmasterminds.com
01.05.2006 01.05.2006 16.06.2005 16.07.2001 01.05.2006 15.06.1997 01.07.1994 16.08.2002 16.05.2008 10.09.2004 16.06.2005 01.07.2003 01.07.2003 12.05.2007 01.09.1997 10.09.2004 16.06.2005 01.05.2006 01.05.2006 01.09.2009 10.09.2004 16.05.2008 16.10.1998 16.07.2001 12.05.2007 01.07.2010 01.07.2010 01.07.2010 01.07.2010 01.07.2010 01.07.2010 01.07.2010 01.07.2010

85. 86. 87. 88. 89. 90. 91. 92. 93. 94. 95. 96. 97. 98. 99. 100. 101. 102. 103. 104. 105. 106. 107. 108. 109. 110. 111. 112. 113. 114. 115. 116. 117.

Share Transfer Agents Services Site Formation & Clearance, excavation, earthmoving & demolition services Sound Recording Services Sponsorship Services Streamer Agents Services Stock Broking services Storage and Warehousing Services Supply of Tangible Goods Survey and Exploration of Mineral Services Survey and Map Making Services Technical Inspection and Certification Services Technical Testing and Analysis Services Telecommunication services Tour Operators Services Transport of Goods by Air Services Transport of Goods by pipeline or other conduit services Transport of Goods in Containers by Rail Services Transport of persons by Cruise Ship Services Transport of goods by water Travel Agents services ULIP Services Underwriters services Video-tape Production Works Contract services Grant of right in relation to an event Maintenance of Medical Records of Employees Medical Services to employees of business entity Preferential location services Promotional services in relation to games of change Promotion of Brand Temporary use of Copyright Trading of electricity

IPCC _27e_Service Tax_Finance Act 2011 ____________________________ 18

No.1 for CA/CWA & MEC/CEC

MASTER MINDS

Q.NO.1. PRIVATE TUITION CENTERS ARE WITHIN THE SCOPE OF LEVY OF SERVICE TAX. EXPLAIN. (OR) COMMERCIAL TRAINING OR COACHING IS AN ACTIVITY WITHIN THE SERVICE TAX AMBIT. DISCUSS. (OR) EXPLAIN THE TERM VOCATIONAL TRAINING INSTITUTE, UNDER THE PROVISIONS OF SERVICE TAX? Effective date

This service is taxable. w.e.f. 01.07.2003 1. Commercial Training or Coaching Centre shall include [Sec.65(26) / 65(27)] Any Centre or Institute, Where training or coaching is imparted for consideration, Whether or not such centre or institute is registered as a Trust or Society or similar other organization, under any law for the time being in force, and Carrying on its activity, with or without profit motive.

Meaning

1. Institutions providing Commercial Coaching or Training, Coaching Classes, Tutorial Classes, Postal Coaching, Home Tuitions by Institutions etc. 2. Commercial Coaching centers, for imparting training to Clients Staff, at Clients premises. 3. Computer Training Institutes (both Hardware and Software). 4. General Course: Institutes, that offer general course, on improving Communication Skills, Personality development, How to be effective in group discussions or Personal interview, General grooming and Finishing etc., are liable to Service Tax. These institutes do not prepare a candidate to take up employment or self-employment directly, but only improve the chances of success, or a candidate, who already has the required skill. [Hence, not covered under Vocational Training Institutes]. Taxable Services 5. Derecognition: An Institution or Establishment, which is derecognized by the professional councils [eg. Medical Council of India (MCI), Bar council of India (BCI), All India Council for Technical Education (AICTE)]. An Institute / establishment may be recognized by the law at one time and may not be recognized at other after getting Derecognition. Service rendered during the derecognized period is taxable. 6. Training / coaching, provided by a Commercial Training or Coaching Centre. 7. Services rendered by a Computer Training Institute 8. Institutions imparting education/ training / coaching skills, helping in preparing for competitive exams or conducting classes, for various subjects. 9. Coaching by postal means is taxable, including the postal charges. 10. If an employer hires an outside commercial coaching or training centre, for imparting some training to its Employees, then the payment made by the said Employer to such coaching centre is chargeable to tax.

IPCC _27e_Service Tax_Finance Act 2011 ____________________________ 19

Ph:

0863 22 42 355
2. Sports Training Institutions.

www.gntmasterminds.com

1. Pre-school coaching and training centre. 3. Individual providing Tuitions at own home. 4. Vocational Training Institute - Means an Industrial Training Institute(ITI) or Industrial Training Centre(ITC) affiliated to the National Council for Vocational Training ,offering courses in designated trades as notified under the Apprentices Act,1961. 5. Commercial training or coaching services, provided by a Vocational Training provider. Condition: He should be registered under the Skill Development Initiative Scheme, with the Director General of Employment and Training, in relation to Modular Employable skill courses, approved by the National Council of Vocational Training. Non Taxable Services 6. Recreational Training Institutes - Commercial Training or Coaching Centre which provides coaching or training, relating to recreational activities, such as dance, singing, martial arts, hobbies, foreign languages, painting hobby classes etc., 7. Institutions issuing any Certificate / Diploma or degree etc., recognized by any law in force. Eg. University / Deemed University, as defined under the UGC Act, Autonomous Colleges as per National Policy on Education, 1986. 8. Commercial Training, which form an essential part of course of curriculum, of any other institute or establishment, leading to issuance of certificate or diploma or degree or educational qualification, recognized by law, for the time, being in force, to any person. 9. Employers providing free training themselves. 10. Individuals providing services at the premises of a Service Receiver are not subject to Service Tax. However, if Coaching or Training Centre provides commercial coaching by sending individuals to the premises of service receivers, such services would be chargeable to tax. 11. Vocational Coaching and Training services provided by typing and shorthand institutes, TV/ Vehicle Repair Training Institutes, Tailoring Institutes, Foreign Language Institutes, Computer Training Centers, Hobby Classes, Institutes Teaching Martial Arts, Painting, Dancing etc., Q.NO.2. MEMBERS OF THE COMMERCE PROFESSION i.e., CHARTERED ACCOUNTANTS, COMPANY SECRETARIES AND COST ACCOUNTANTS ARE LIABLE TO PAY SERVICE TAX FOR THE SERVICES RENDERED BY THEM. DISCUSS. [SEC 65 (105) (U), 65 (105) (T)]

Meaning

1. Practicing Chartered Accountants [Sec 65(83)]: Members of the Institute of Chartered Accountants of India, holding the Certificate of Practice. 2. Practicing Company Secretaries [Sec 65(85)]: Members of the Institute of Company Secretaries of India, holding the Certificate of Practice.

IPCC _27e_Service Tax_Finance Act 2011 ____________________________ 20

No.1 for CA/CWA & MEC/CEC

MASTER MINDS

3. Practicing Cost Accountants [Sec 65(84)]: Members of the Institute of Cost and Works Accountants of India, holding the Certificate of Practice. Persons liable to pay tax Independent Practitioners and Firms of Practicing Chartered Accountants, Practicing Company Secretaries and Practicing Cost Accountants. 1. All services, provided by Practicing Chartered Accountants, Practicing Company Secretaries and Practicing Cost Accountants, in their professional capacity, in any manner. 2. Services, in relation to representation before Statutory Authorities, if such representations are in the course of proceedings initiated under any law, for the time being in force, by way of issue of notice. Services relating to Teaching, Placement etc. (not deemed to be provided in professional capacity).

Taxable Services

Non-taxable Services

Q.NO.3. SERVICES RENDERED BY PROFESSIONAL ENGINEERS ARE LIABLE TO SERVICE TAX. EXPLAIN. [SEC 65(105)(G)]

1. Consulting Engineer refers to [Section 65(31)] a. Qualification: Professionally Qualified Engineer or any other Body Corporate or any other Firm; Meaning b. Service: Rendering advice, consultancy or technical assistance, in any manner to any person. c. Area: In any discipline of Engineering, including Computer Hardware Engineering and Computer Software Engineering. 2. Taxable Service [Section 65(105)(g)] Service provided to any person, by a Consulting Engineer, in relation to advice, consultancy or technical assistance, in one or more disciplines of engineering. 1. Includes: Consulting Engineers, Engineering Firms, Project Consultants, Firms engaged in supply of Designs, Drawings, Technical Assistance. 2. Excludes: a. Individuals, not holding any degree or Diploma in Engineering, rendering Consulting Engineering Service. b. Firms engaged in Post Sale Services. c. Engineers dealing with sea worthiness of ships. 1. Services provided, in the discipline of computer hardware engineering and computer software; Taxable Services 2. Services of Valuation. 3. Soil Testing, Land Surveys, Structural Engineering Works, Civil, Mechanical or Electrical engineering, Construction Management, Survey, Planning, Supply of Drawings and Designs etc. 4. Services provided by self employed, professionally qualified engineer.

Persons liable to pay tax

IPCC _27e_Service Tax_Finance Act 2011 ____________________________ 21

Ph:

0863 22 42 355

www.gntmasterminds.com

1. Testing laboratories, Supply of marine crafts, Piloting of vessels from anchorage, Pollution Control Steps, Overseas Project. 2. Services of certification or inspection carried out as legal requirement, under a statute or code [Technical Inspection and Certification Service]. 3. Services of Soil Testing (for Mothilal Nehru Institute only). Non-Taxable Services 4. Supply of Technical Manpower [Manpower Recruitment and Supply Agency Services]. 5. Works Contracts Service. 6. Commissioning, Erection & Installation etc. 7. Payment of royalty for use of Technical Know How, Brand Name, Trade Mark, Patent etc.; [Intellectual Property Service]. 8. Providing training to customers for correct use of technology. Specific Exemption Exemption for the amount of Cess paid Under Sec 3 of R & D Cess Act, 1986, from the amount of Service Tax payable, under Consulting Engineers Service.

Q.NO.4. CHARTERED ACCOUNTANT PLAYS A SIGNIFICANT ROLE, IN THE MATTERS INVOLVING SERVICE TAX EXPLAIN. 1. Compliance: The various procedures relating to Registration, payment of Tax, returns and Assessments are to be complied with, and hence a Chartered Accountant, with his expertise, can facilitate such compliance. 2. Representations: CAs can represent their Clients before the various Service Tax assessment Authorities, Commissioner (Appeals) and Tribunals. 3. Continuous Updation: There is an increasing requirement for professionals, to update themselves with the latest developments, because the Service Tax provisions are governed mainly by way of Notifications, issued by various Commissionerates. 4. Advisory Services: Various provisions of Service Tax requires an indepth interpretation and analysis, and hence, there is a demand for professional advice among service providers. 5. Wider Scope: With rapid growth of Service Sector, the scope and potential for practice is also expanding. Q.NO.5. INFORMATION TECHNOLOGY SERVICES ARE SUBJECT TO SERVICE TAX. EXPLAIN. Effective date Meaning

This service is taxable w.e.f 16.05.2008 1. Taxable Services [Sec.65(105) (zzzze] Information Technology Software Service shall be taxable even if they are not provided for use in the course, or furtherance, of business or commerce.

IPCC _27e_Service Tax_Finance Act 2011 ____________________________ 22

No.1 for CA/CWA & MEC/CEC


2. Information Technology Software [sec.65(53a)]:

MASTER MINDS

a. Content: Any representation of instructions, data, sound or image, including source code and object code. b. Format: Such content should be recorded in a machine readable form. c. Usage: Software should be capable of being manipulated or providing interactivity to a user, by means of a computer or an automatic data processing machine or any other device or equipment.
Any person providing (w.r.e.f.16.05.2008) services in relation to Information Technology is liable to pay tax. The following are the services in relation to Information Technology Software 1. Development. 2. Study, Analysis, Design and Programming. 3. Adaptation, Upgradation, Enhancement, Implementation and other similar related services. Taxable Services 4. Providing advice, consultancy and assistance on matters related to information technology software, design, guidance and assistance during the startup phase of a new system, specifications to secure a database, advice on proprietary information technology software. 5. Providing the right to use information technology software for commercial exploitation including right to reproduce, distribute and sell information technology software and right to use software components for the creation of and inclusion in other information technology software products. 6. Providing the right to use information technology software supplied electronically. Services provided to individuals for use, other than in business or commerce, such as for personal use, are not taxable under Information Technology Software services. Taxable service providing Packaged or Canned Software, intended for single use and packed accordingly shall be exempt subject to the following conditions 1. Document providing right to use such software is packed along with the software, 2. The Importer has paid the appropriate duties of customs or in case of Manufacturer, duplicator, or the person holding the copyright to software, he has paid the appropriate duties of excise on the entire amount received form the buyer, and 3. The benefit under any other relevant notification is not availed by him.

Nontaxable Services

IPCC _27e_Service Tax_Finance Act 2011 ____________________________ 23

Ph:

0863 22 42 355

www.gntmasterminds.com

Q.NO.6. WHO ARE LIABLE TO PAY SERVICE TAX UNDER SCIENTIFIC OR TECHNICAL CONSULTANCY SERVICE? [SECTION 65(105)(ZA)] 1. Scientific or Technical Consultancy means [Section 65(92)] a. Activity: Any Advice, Consultancy, or Scientific/ Technical Assistance provided to a client; b. Person: By a Scientist or a Technocrat, Science / Technology Institution/ Organization; c. Scope: In one or more disciplines of science or Technology. 2. Taxable Service [Section 65(105)(za)] ___ Service provided to any person by a Scientist, or a Technocrat or any Science or Technology Institution or Organization in relation to scientific or technical consultancy. 1. Inclusions: a. Scientist, Technocrat, b. Energy Consultants, Rehabilitation Consultants c. Educational and Research Institutes. d. Public/Private Funded Research Institutions such as: Persons liable to pay Tax Council of scientific and Industrial Research(CSIR) Indian Council of Agricultural Research(ICAR) Defence Research and Development Organization(DRDO) Indian Institute of Technology(IIT) National Institute of Engineering Colleges) Technology (NIT) (formerly Regional

Meaning

Indian Institute of Science(IISC) 2. Exclusion: Diagnostic and Pathological Labs, Doctors, Medical Colleges, Nursing Home, Hospitals 1. Services such as complete technical information and training of employees about the projects relating to research and development activities with regard to automotive tyres. (CCE v.MRF Ltd.2005149 ELT 472) 2. Services rendered to Government departments, public sector undertakings. 3. Testing services which are integral part of the Scientific or Technical Consultancy 1. Activity of in house development of technology, which is sold by a company to its sister concern. Not a scientific or Research Institution. Hence, not taxable. [Rubco Sales International Pvt.Ltd.2006 TIOL 188(CESTAT)] 2. Testing Services are not covered, unless it is an integral part of consultancy (Technical Testing and Analysis Service) 3. Research/Projects undertaken by Research Institution receiving Grants/ Aids (No service is rendered to the Donor) Service by a Foreign Technician visiting India Taxable as the services are provided in India.

Taxable Services

NonTaxable Services

IPCC _27e_Service Tax_Finance Act 2011 ____________________________ 24

No.1 for CA/CWA & MEC/CEC

MASTER MINDS

Q.NO.7. EXAMINE THE APPLICABILITY OF SERVICE TAX LAW TO TECHNICAL TESTING AND ANALYSIS SERVICE. [SEC 65(105)(ZZH)] Technical Testing and Analysis means [Section 65(106)-1. Activity: Service in relation to physical, chemical, biological or any other scientific testing or analysis Meaning 2. Scope: Testing/Analysis ofa. Inclusion: Goods or Material or any Immovable Property, w.e.f.16.05.2008 including Information Technology Soft ware b. Exclusion: In relation to Human Beings/Animals Persons liable to pay tax Taxable Services Technical Testing and Analysis Agency [Section 65(107)], Sample Collection Centre, Clinical Research Organization 1. Clinical testing of Drugs and Formulations. 2. Water Testing by other than Government Laboratories. 1. Testing or analysis service provided in relation to human beings or animals i.e. Medical Testing and Diagnosis 2. Services provided while determining the nature of disease, identification, prevention of disease or disorder in human beings or animals. 3. Services in relation to water quality testing by Govt owned state or District level laboratory. 1. Technical Testing and Analysis of newly developed drugs (including herbal remedies and vaccine) on human participants, by Clinical Research Organisation (CRO). [ Notfn.No.6/2006] 2. Services provided by a Government owned State or District Level Laboratory in relation to testing and analysis of water quality [Notfn.No.6/2006] Dr. Lal Path Lab(P) Ltd.2007(008) STR 0337 P &H

Non-taxable Services

Specific Exemption

Activity of a collection Center for drawl of blood samples and to carry out essential processing (serum separation) of blood and forwarding samples to the principal lab through courier is not covered under Technical Testing and Analysis Service since Service are in relation to Human Beings.

Q.NO.8. EXAMINE THE APPLICABILITY OF SERVICE TAX LAW TO BUSINESS EXHIBITION SERVICES. Effective date

W.e.f.10.09.2004 as per Finance Act, 2004 Business Exhibition means an exhibition,_____ a) to market; or b) to promote; or c) to advertise; or

Meaning

d) to showcase,

any product or service, intended for the growth in business of the producer or provider for such product or service, as the case may be

IPCC _27e_Service Tax_Finance Act 2011 ____________________________ 25

Ph:

0863 22 42 355
to an exhibitor,

www.gntmasterminds.com

1. Taxable service means any service provided or to be provided: by the organizer of a business exhibition, in relation to business exhibition 2. Meaning of organizer of business exhibition: The term Organizer has not been defined. However, it should be understood in the common parlance. In common parlances, organizer means a person / firm / company that provides space or may be in addition provides furniture, cabins, security, electricity, etc. and solicit the participation from trade and industries and charge their customers accordingly. 3. Meaning of Exhibitor: The term exhibitor has not been defined but it should not be accorded a narrow meaning. Any person desirous of displaying his product or service could be termed as an exhibitor. 4. Types of activities covered under Business Exhibition Services: Trade fairs, Road shows. Fashion shows, display show cases kept in airports, railway stations, and hotels are covered under this category. Taxable Services Thus the organizers of trade fairs and exhibitions provide services in relation to business exhibition. Such services is liable to service tax and covered under business exhibition service and not under Event Management Services. The organizer may engage an event manager to provide service in relation to organizing trade fairs and exhibitions but in that case, the service rendered by the event manager are in relation to planning, promoting, organizing, etc. The service rendered by the event manager to the organizers, is liable to service tax under Event Management Service. Similarly, Campaigns and road shows organized for promotion of products are covered under business exhibition services and not under event management services. 5. Difference between Advertisement Service and Business Exhibition Service: Services rendered in relation to a circular, label, document, hoardings or any other audiovisual representation of a product or service falls under Advertisement Services whereas the service relating to actual exhibition or display of the product or services would fall under the category of Business Exhibition Services. 6. The value of taxable service: The value of taxable service shall be the gross amount charged by the organizer of such business exhibition from an exhibitor (including amount claimed as reimbursements). Non Taxable Services The services, which are not covered under this category of taxable service and / or specifically exempted (they might be covered under other category of taxable service), are given below:---

IPCC _27e_Service Tax_Finance Act 2011 ____________________________ 26

No.1 for CA/CWA & MEC/CEC

MASTER MINDS

1. Display of consumer goods in shops or shopping centers by the shop keepers for displaying such goods for customers, to select and purchase, will not be covered under this taxable service, as normally no separate charge are collected by the shopkeepers for displaying such goods. However, in case any amount is collected for merely displaying an item, the same would be chargeable to service tax. 2. Services provided by an organizer of business exhibition for holding a business exhibition out side India. Such service will be exempt from the whole of service tax Q.NO.9. EXAMINE THE APPLICABILITY OF SERVICE TAX LAW TO MANDAP KEEPERS SERVICES. Effective date

W .e. f. 1-7-1997, Notification 19/97-ST, dated 26-6-1997. 1. Meaning of mandap keeper [Section 65(67)]: Mandap keeper means a person who allows temporary occupation of a mandap for a consideration for organizing any official, social or business function. Explanation: For the purpose of this clause. social function includes marriage. The program of dance, drama or music is a social function and hence covered under the definition ofmandap keeper service. Thus renting out hall etc. for such purpose shall be chargeable to service tax. Mandap keeper may not be owner of the premises: It is not necessary that the mandap keeper must be owner of the premises. Even a person who is not an owner of the premises would be liable for service tax if he keeps the premises and is involved in specified activities. For example, many halls are given to particular persons on a monopoly basis on a rental for the entire year. Such monopoly holders then let out the premises for various events. In such situations, the monopoly holder is liable for services tax under the category of mandap keeper and not the actual hall owner. 2. Meaning of mandap: mandap means any immovable property as defined in section 3 of the transfer of property Act, 1882 and includes any furniture, fixtures, light fitting and floor coverings there in let out for a consideration for organizing any official, social or business function. The definition of mandap is very wide in its coverage and covers all immovable properties let out for organizing social, official or business functions. It includes within its scope places like Kalyana mandaps or marriage halls, banquet halls, conference halls, etc. Hotels and restaurants providing any such facility within their premises for organizing any social, official or business function shall also be obviously included in the coverage of service tax. Since a boat is not an immoveable property, any service of letting out of boats for organizing boat parties or events on boats would not be liable for service tax.

Meaning

IPCC _27e_Service Tax_Finance Act 2011 ____________________________ 27

Ph:

0863 22 42 355

www.gntmasterminds.com

3. Meaning of caterer [Section 65(24)]: Caterer means any person who supplies, either directly or indirectly, any food, edible preparations, alcoholic or non-alcoholic beverages or crockery and similar articles or accoutrements for any purpose or occasion. 1. Taxable Service means any service provided or to be provided to ----- any person, by a mandap keeper in relation to the use of mandap in any manner Including the facilities provided or to be provided to such person in relation to such use and also the services, if any, provided or to be provided as a caterer. 2. Scope of taxable service: The scope of taxability is expanded to include all facilities provided to the client in relation to such use (like electricity, furniture, etc.) and also services rendered as a caterer. In other words, the service tax would fall not only the hire charges for the mandap but also, charges for electricity, whether on actual basis or otherwise, charged to the customer. Therefore, it may be ensured that the tax is collected on the whole amount even if separate bills are issued, one for the rental and the other for electricity charges. The mandap keepers may also bill the client for other rendered by him such as charges for providing furniture, fixtures, lighting fitting, vessels, crockery, cutlery, etc. All these charges are includible in the value of taxable service for levy of service tax. Where a mandap keeper also provides catering services, that is, supply of food, in addition to the letting out of the mandap and charges the customer for supply of goods, an abatement of 40% of the total amount charged has been provided while computing the value of the taxable service The charges of vessels, furniture, decoration etc. Provided by the third party other than the mandap keepers are not includible in the taxable if the mandap keepers is not associated with such supply in any way since the said facility is not provided by the mandap keeper The service, which are not covered under this category of taxable service and/or specifically exempted (they might be covered under other category of taxable service), are given below: 1. Mandap keeper service provided by churches, temples, gurudwaras, etc. in their religious precinct for hosting of social and religious furnishing are exempt. However, w.e.f. 1.6.2006 exemption of service provided by mandap keeper in relation to use of mandap for organizing marriage function has been withdrawn. 2. Renting out of premises by art galleries to artists, painters, sculptors, to exhibit their works of art will not be liable to service tax under the category of mandap keepers service. 3. Reservation of seats in a restaurant is not taxable service because service rendered by mandap keeper is taxable only when he has let out some rooms, halls etc. and essentially hand over the temporary possession to the person to whom it is let out but reservation of seats does not include any such service.

Taxable Services

Non Taxable Services

IPCC _27e_Service Tax_Finance Act 2011 ____________________________ 28

No.1 for CA/CWA & MEC/CEC

MASTER MINDS

4. Sales tax, expenditure tax, or any other tax charged in the bill will be excluded from value of taxable service or business function. 5. Screening of feature films in cinema theatre is not taxable because, this is not an official, social or business function. 6. If a mandap keeper provides catering service in a composite manner along with service in relation to mandap keepers service, i.e. supply of food and the invoice, bill or challan issued indicates that is is inclusive of the charges for catering services; tax is leviable only to the extent of 60% of the consolidated value received by mandap keeper as in this case 40% abatement shall be allowed. However, the abatement shall be allowed if both the following conditions are satisfied: a. The CENVAT credit of duty paid on inputs or capital goods or the CENVAT credit of service tax on input service , used for providing such taxable service has not been taken; and b. The service provider has not availed the benefit under the notification No. 12/2003-ST, dated 20.06.2003

Illustrations
1. ABC Associates is a commercial training centre. It was established in 2005 and since then, the Gross annual receipts are more than Rs. 4,00,000 every year. It charges Rs. 1,50,000 per student per batch for providing training. The break up of the bills is as followsTraining charges Rs. 1,20,000 Value of books sold Rs. 30,000 Compute the amount of Service Tax to be charged. Solution: Computation of Service Tax liability Particulars Training charges Less: Value of books sold (exempted under Notification No. 12/2003 ST) Total value of taxable service Service Tax @ 10.3% Amount 1,20,000 Nil 1,20,000 12,360

2. Commercial Training or Coaching Services: JB Institute of Management is engaged in providing vocational training to the students. The services provided are taxable under the category of Commercial Training or Coaching Services Explain the validity of the statement. Solution: The statement is invalid. Notification No. 24/2004, dated 10.09.2004 provides that the taxable services provided to any person in relation to commercial training or coaching, by a vocational training institute are wholly exempt from the Service Tax. Therefore, in the given case, services provided by JB Institute of Management are not taxable under the category of Commercial Training or Coaching Services.

IPCC _27e_Service Tax_Finance Act 2011 ____________________________ 29

Ph:

0863 22 42 355

www.gntmasterminds.com

3. Smart & Express Co., is providing taxable information Technology software services. The firm furnishes the following information relating to the services rendered, bills raised, amount received pertaining to this service, for the financial year ended on 31st March, 2012 as under: Particulars I. Amount received being 10% of the assignment fees on 31st March, 2012 for the upgradation and enhancement of software services to be rendered during the financial year 2012-2013. Rs. Rs. 6,00,000

II. Services provided to UNICEF, an international Organsiation in Gandhinagar, for analysis, design, and programming of latest information technology software. III. Services billed to clients (In one of the bill amounting to Rs.3,00,000 Service Tax was charged due to conflicting nature and in another bill the firm failed to recover the Service Tax from the client, which was charged separately, due to insolvency of the client, the bill details are as under: Being the charges for right to use IT software Service Tax @ 10% Education cess @ 2% Secondary & Higher education cess @ 1% IV. Amount received for services rendered after the issue of invoice during current financial year (excluding payment for 2 bills in item (III) above for which payment received during current financial year, invoice is issued within 14 days from the completion of service).

5,00,000

3,00,00,000

8,00,000 80,000 1,600 800 8,82,400 1,04,78,500

Service Tax and Education Cess have been charged separately in all the bills except wherever mentioned when it is not so charged separately. Compute the value of total taxable services and Service Tax payable thereon for the year ended 31-3-2012, assigning reason in brief to the treatment of at items. Solution: Assessee: Smart & Express & Co. Computation of value of taxable services and Service Tax Payable for the year ended 31.03.2012 Particulars I. Amount received being 10% of the assignment fees on 31st March, 2012 for the upgradation and enhancement of software services to be rendered during the financial year 2012-2013. Taxable Value Reasons Adaptation, upgradation, enhancement, implementation and other similar activities are taxable services.

5,43,971

IPCC _27e_Service Tax_Finance Act 2011 ____________________________ 30

No.1 for CA/CWA & MEC/CEC


II. Services provided to UNICEF, an International Organsiation in Gandhinagar, for analysis, design and programming of latest information technology software. III. Services Billed to Clients (2,89,00,000 + 3,00,000+ 8,00,000) *100/110.3) 2,71,98,549

MASTER MINDS
All taxable services provided by any person to the United Nations or an International Organisation are exempt from whole of Service Tax. Though the service provided charges Service Tax in his bill raised on his client as and is payable to the Government only when the payment is received or invoice issued within 14 days whichever is earlier.

NIL

Taxable value of Services Service Tax Payable @ 10% Add: Education Cess @ 2% of Service Tax Add: SHEC @ 1% of Service Tax Total Service Tax Payable

2,77,42,520 27,74,252 55,485 27,743 28,57,480

4. Mr.Ajay, a chartered accountant represents his client Mr.Vijay in a case relating to evasion of Income Tax before the Income Tax Appellate Tribunal. Mr.Ajay charges Rs.25,000 as consideration for this service. Discuss whether Mr.Ajay is liable to pay Service Tax on such service. Solution: a. In the given case, Mr.Ajay is representing his client Mr.Vijay before the Income Tax appellate Tribunal, a statutory authority, under a proceeding initiated under Income Tax Law. b. Thus, in view of the provisions of the Service Tax Law in relation to Chartered Accountants Services, Mr.Ajay shall be liable to pay Service Tax on the services rendered by him. 5. State, with reason in brief whether the following statement is True or False, with reference to the provisions of Service Tax: Mr.Salim, an Architect, has received the fees of Rs.4,48,500 after deduction of Income Tax of Rs. 51,500. The Service Tax payable is on Rs. 4,48,500. Solution: a. TDS is a part of Taxable Value: Service Tax is to be paid on the value of taxable service, which is charged by a Service Tax assessee. Income Tax, deducted at source, is included in the charged amount. Service Tax is, therefore, payable on the total amount, inclusive of the Income Tax, deducted at source. b. Conclusion: Hence, the given statement is False. Service Tax is payable on Rs. 4,48,500 + Rs.51,500 = Rs. 5,00,000.

IPCC _27e_Service Tax_Finance Act 2011 ____________________________ 31

Ph:

0863 22 42 355
Self Test Questions

www.gntmasterminds.com

1. Discuss whether the following persons are liable for registration under Service Tax. If yes, when is the registration required? a. A provider of taxable services, whose aggregate value of taxable services is Rs. 8,00,000, up to December 31st, 2011. b. An input service distributor, who starts his business on January 1st, 2012. c. A provider of taxable services, under a brand, which is not yet registered, provided services of Rs.8,00,000, up to January 31st, 2012. 2. Commercial training or coaching services: With reference to commercial training or coaching services, state whether Service Tax is applicable in the following cases: a. Pinnacle Institute offering courses on personality development and grooming b. BTL Engineering College, offering B.Tech to students. However, the college has been derecognized by the All India Council for Technical Education. 3. Commercial training or coaching services: XYZ is an educational institute. It provides commercial training for imparting knowledge on commerce and accountancy without issuance of any certificate. Will it be liable to Service Tax? Will your answer be different if certificates are issued for the same? 4. State, whether the following service is taxable under the provisions of the Finance Act, 1994, relating to Service Tax: A practicing Chartered Accountant representing a client before Income-tax Officer in assessment proceeding. 5. When was the service provided by a practicing Chartered Accountant brought within the purview of Service Tax? 6. What are the due dates for filing of returns under the Services Tax Law? 7. Does the doctrine of unjust enrichment apply to payments under the Service Tax Law? 8. A practicing Chartered Accountant also provides manpower recruitment services to his client. Is this is a taxable service? 9. Cargo Handling Services in relation to Agricultural Produce and Cold Storage. Explain whether the service is taxable under the provisions of the Finance Act, 1994, relating to Service Tax? 10. Services by a consulting engineer relating to computer software. Explain whether the service is taxable under the provisions of the Finance Act, 1994, relating to Service Tax?

IPCC _27e_Service Tax_Finance Act 2011 ____________________________ 32

No.1 for CA/CWA & MEC/CEC

MASTER MINDS

PART 3. VALUATION OF SERVICE TAX AND ITS RULES


NO. 1. 2. 3. QUESTION Explain the valuation of Taxable Service for the charging of Service Tax. When is the value of materials sold, in provision of service, allowed as deduction? Value of taxable service cannot be determined, when the whole of the consideration is in non-monetary form. Comment on the validity of this statement. What are the considerations in the determination of taxable value of services? (Or) Will the payment to a hotelier of Rs.10,000, on behalf of an architect by a service receiver, be included in the value of taxable services? Write about Service Specific Inclusions. What are the service specific exclusions in the valuation of taxable service? Write a note on the Service Tax (Determination of Value) Rules, 2006 What are the conditions incidental to the payment of Service Tax? Is the Service Tax payable by the assessee, even in cases where his clients do not pay for the service rendered or pay only a part of the bill raised in this regard? ABC A B A

4.

B C C A B

5. 6. 7. 8.

Q.NO.1. EXPLAIN THE VALUATION OF TAXABLE SERVICE FOR THE CHARGING OF SERVICE TAX. As per Section 66, Service Tax is to be charged @ 10% of the value of taxable service provided or to be provided. Hence, we have to determine the value of such taxable services, so as to pay Service Tax. If the consideration consisting of: a. Where the provision of service is for a consideration in money: The gross amount charged by the service provider for such service provided or to be provided by him. Such amount in money, with the addition of Service Tax charged, as is equivalent to the consideration. In other words, it should be of the value of similar services, provided to a third party. If similar service is not provided to a third party, it should be the market value as determined by the assessee but it cannot be lower than the cost of provision of the same. The amount, as may be determined in the prescribed manner.

b.

Where the provision of service is for a consideration, not wholly or partly, consisting of money:

c.

Where the provision of service is for a consideration, which is not ascertainable:

IPCC _27e_Service Tax_Finance Act 2011 ____________________________ 33

Ph:

0863 22 42 355

www.gntmasterminds.com

a. Consideration also includes any amount, which is payable for the taxable services provided or to be provided. b. Money includes any currency, cheque, promissory note, letter of credit, draft, pay order, travellers cheque, money order, postal remittance and other similar instruments, but does not include currency that is held for its numismatic value. c. Gross amount charged include payment by cheque, credit card, deduction from account and any form of payment by issue of credit notes or debit notes and book adjustment. d. The first basic principle is that, Service Tax is payable on the gross amount, and not on the net amount. e. The second principle, which also flows from Section 67, is that the amount charged has to be in relation to the services rendered. f. Valuation, where billing is inclusive of Service Tax: The assessee is to calculate the Service Tax, based on reverse working. Eg.: If the billing is done inclusive of Service Tax [Consider billing 100 (inclusive), realization 100 (inclusive)] the assessee can claim that the bill amount should be bifurcated into value and Service Tax components. Accordingly the Service Tax payable shall be (100 10.3/110.3) = Rs. 9.3381. g. Payment through Suspense Account: In case of transactions with associated enterprise, payments received towards the value of taxable services shall include any amount credited or debited, to any account, whether called Suspense Account or by any other name, in the books of account of a person liable to pay Service Tax. Q.NO.2.WHEN IS THE VALUE OF MATERIALS SOLD, IN PROVISION OF SERVICE, ALLOWED AS DEDUCTION? 1. Material Value: a. Excluded: If the service provider supplies goods and material while providing the service, value of goods and material will be excluded, for the purpose of payment of Service Tax. b. Value vs. Cost: The amount deductible is the value of the materials/goods supplied, and not the cost. Value refers to the worth of a thing in money, whereas cost is the payment required for acquiring such material. Example: In case of commercial training and coaching institutes, deduction will be available only to the sale value of standard text books, which are priced. Any study material or written text, provided by the institute, will not be allowable as deduction. 2. Deduction for materials sold: If the amount charged includes value of goods and materials sold, Service Tax will not be payable on value of goods and materials sold, provided: a. Sale of such goods is evidenced and sale value is quantified, and shown separately in invoice. b. There is documentary proof, specifically indicating the value of the said goods and materials. c. The service provider has not claimed any abatement / exemption benefit, under any notification, and Service Tax is paid at full rate.

IPCC _27e_Service Tax_Finance Act 2011 ____________________________ 34

No.1 for CA/CWA & MEC/CEC

MASTER MINDS

3. Treatment of materials consumed: Materials consumed in providing a service cannot be reduced from value, for the purpose of measuring Service Tax payable. Deduction is available only for the value of goods and materials sold, subject to maintenance of documentary evidence of such sale. For example, cost of materials used for cosmetics in beauty parlour or Soap oil in repair and maintenance service cannot be excluded from taxable value as they are consumed and not sold. Q.NO.3.VALUE OF TAXABLE SERVICE CANNOT BE DETERMINED, WHEN THE WHOLE OF THE CONSIDERATION IS IN NON-MONETARY FORM. COMMENT ON THE VALIDITY OF THIS STATEMENT. When the whole of consideration received for provision of service is in non-monetary form, Taxable Value = Money Equivalent to non-monetary consideration, as determined by the service provider, as under Value of similar service is available Taxable value of service = Gross amount charged by the service provider to provide similar service to any other person in the ordinary course of trade. Notes: 1. Similar Service: a. Meaning: Similar does not mean identical but it means corresponding to or resembling in many respects. It does not mean same in all respects. b. Criterion: Where the Service Tax is charged on the basis of similar services provided by the same person, the same should be based on a: Normal transaction, Between two independent persons (i.e. unrelated parties) & At an Arms length price. 2. Cost + Reasonable Profit: The service provider may ascertain the cost of provision of service based on usual costing principles of normal costs and allocation of normal overheads. A reasonable profit should be added to the cost so determined to ascertain the value of taxable service. However, the value so determined should not be less than the cost of provision of such service. Q.NO.4. WHAT ARE THE CONSIDERATIONS IN THE DETERMINATION OF TAXABLE VALUE OF SERVICES? (OR) WILL THE PAYMENT TO A HOTELIER OF Rs.10,000, ON BEHALF OF AN ARCHITECT, BY A SERVICE RECEIVER BE INCLUDED IN THE VALUE OF TAXABLE SERVICES? (OR) WHETHER FREE SERVICES, AFTER SALE OF MOTOR VEHICLES, GIVEN BY THE AUTHORISED DEALERS, FOR WHICH THEY ARE REIMBURSED BY THE VEHICLE MANUFACTURERS, SUBJECT TO SERVICE TAX? Value of similar service is not available Taxable value of service = Cost of provision of service + Reasonable profit.

IPCC _27e_Service Tax_Finance Act 2011 ____________________________ 35

Ph:

0863 22 42 355

www.gntmasterminds.com

1. Service purpose Vs other purpose: Service Tax is payable on taxable value of services i.e., the gross amount charged by the service provider, for the service rendered by him. Amount charged for some other purpose, other than for provision of service, will not be included in taxable value. Example: Interest on delayed payment. 2. Expenditure/costs: Expenditure or costs incurred by the service provider, in the course of providing taxable service:
Shall be treated as consideration for the taxable services provided or to be provided,

even if such costs or expenditure are recovered separately, by the service provider. 3. Method of billing: (Not relevant): Value of taxable services is the total amount of consideration consisting of all components of the taxable service, irrespective of the fact that details of individual components of the total consideration are indicated separately in the invoice. 4. Costs directly incurred by the service receiver or out-of- pocket expenses: Out-ofpocket expenses (reimbursable separately by the service receiver), incurred by the service provider in the course of providing the service, is includible in the taxable value. Example: Travelling, Hotel, Telephone expenses, etc. 5. Reimbursement: Taxable value includes cost/expenditure incurred by the service provider and recovered from the service receiver. 6. Warranty Period: Service Tax is leviable on service provided during warranty period, even though the customer is not charged any amount. This is because, the amount is paid directly to the service provider, by the manufacturer of automobile/equipment. 7. Charge on anyone: Amount may be charged to anyone, not necessarily to service receiver. Even if the amount is charged to some other person, and not to the receiver of service, Service Tax becomes payable. Example: Authorised service station services service during warranty period, rendered to the vehicle owner Consideration for the same is paid by the manufacturer of vehicles. 8. Consideration of third parties: It is not necessary that the money should be paid to the service provider himself. Amount paid to a third party is includible in value of service, if

a. It is for provision of service, and b. At the instance of the service provider


9. Time of receipt of consideration: Consideration for service provided may be received at any time, i.e., before, during or after, provision of service. 10. Composite Service: A service has to be pre-dominantly taxable service, for the levy of Service Tax. Where a composite service, consists of some part which is taxable and some part which is not taxable, Service Tax cannot be levied on non taxable services. A composite contract cannot be vivisected and service portion cannot be subjected to tax Widia GMBH (2006) 5 STT 414 (CESTAT)

11. Indivisible Contracts: In case of indivisible contracts, involving both sale of goods and provision of services, Service Tax can be imposed, only on value of service and not on the material cost at all.

IPCC _27e_Service Tax_Finance Act 2011 ____________________________ 36

No.1 for CA/CWA & MEC/CEC

MASTER MINDS

12. Service Tax not paid by recipient: Service Tax liability will still be there, even if the Service Tax is not paid by the service recipient. Q.NO.5. WRITE ABOUT SERVICE SPECIFIC INCLUSIONS. Service Specific Inclusions [Rule 6(1)) The items to be included in the taxable value of service, which are specific to the concerned service are: Service Classification Stock Broking Services Telecommunication Service Cost/Expenditure/Commission included Commission/brokerage, charged by a broker, on the sale or purchase of securities, including the commission or brokerage paid by the stock-broker, to any sub-broker. Adjustments made by telegraph authority, from any deposits made by the subscriber, at the time of application for Telephone Connection / Pager/ Facsimile / Telegraph / Telex / Leased Circuit, etc. Amount of premium, charged by the insurer, from the policy holder; Commission received by the Air Travel Agent, from the airline; Commission, fee or any other sum: a. Received by an Actuary or Intermediary or Insurance Intermediary or Insurance Agent, from the insurer; b. Paid to such an agent, by the Insurer appointing such an agent, in relation to Insurance Auxiliary Services, provided by an Insurance Agent; Authorized Service Station Reimbursement received by Authorized Service Station, from the manufacturer, for carrying out any service of any motor car, light motor vehicle or two wheeled motor vehicle manufactured by such a manufacturer; Commission or any amount received by the Rail Travel Agent, from the Railways or the Customer; Remuneration or Commission, by whatever name it is called, paid to such an agent by the client engaging such an agent, for the services provided by a clearing and forwarding agent, to a client, rendering services of clearing and forwarding operations in any manner.

Insurance Services (Life and General) Air Travel Agent Insurance Auxiliary Services

Rail Travel Agent Clearing and Forwarding Agency

Q.NO.6. WHAT ARE THE SERVICE SPECIFIC EXCLUSIONS IN THE VALUATION OF TAXABLE SERVICE? The value of any taxable service does not include the following: 1. Initial deposit made by the subscriber at the time of application for telephone connection or pager or facsimile (FAX) or telegraph or telex or for leased circuit;

IPCC _27e_Service Tax_Finance Act 2011 ____________________________ 37

Ph:

0863 22 42 355

www.gntmasterminds.com

2. Airfare collected by the Air Travel Agent 3. Rail fare collected by Rail Travel Agent 4. Interest on loans 5. Amount collected for delayed payment of a telephone bill not treated as consideration charged for provision of telecom service and hence not a part of the value of taxable service. 6. Taxes levied by any Government on any passenger traveling by air, if shown separately on the tickets or in the invoice, issued for such ticket. (w.e.f. 27-02-2010) Q.NO.7. WRITE A NOTE ON THE SERVICE TAX (DETERMINATION OF VALUE) RULES, 2006. The provisions of the, Service Tax (Determination of Value) Rules, 2006, are as follows: 1. Determination of Value when consideration is wholly or partly not in money [Rule 3]: Where the consideration received is not wholly or partly consisting of money, the value shall be determined by the service provider, in the following manner: a. the value of such taxable service shall be equivalent to the gross amount charged by the service provider to provide similar service to any other person, in the ordinary course of trade and the gross amount charged is the sole consideration; b. Where the value cannot be determined in accordance with Clause (a), the service provider shall determine the equivalent money value of such consideration, which shall, in no case, be less than the cost of provision of such a taxable service. 2. Rejection of Value [Rule 4] : The Central Excise Officer has the power to satisfy himself as to the accuracy of any information, furnished or document presented, for valuation. Where the Central Excise Officer is satisfied that the value, so determined by the service provider, is not in accordance with the provisions of the Act or the rules, he shall issue a notice to such a service provider to show cause why the value of such a taxable service for the purpose of charging Service Tax should not be fixed at the amount specified in the notice. After providing a reasonable opportunity of being heard, the Central Excise Officer shall determine the value of such taxable service, in accordance with the provisions of the Act and these rules. 3. Inclusion in or exclusion from value of certain expenditure or costs [Rule 5] : , Where any expenditure or costs are incurred by the service provider, in the course of providing taxable service, all such expenditure or costs shall be treated as consideration, for the taxable service provided or to be provided and shall be included in the value for charging Service Tax on that service. Expenditure or costs incurred as 'pure agent' to be excluded: The expenditure or costs incurred by the service provider, as a pure agent of the recipient of service, shall be excluded from the value of the taxable service, if all the following conditions are satisfied: a. the service provider acts as a pure agent of the recipient of service, when he makes payment to a third party for the goods or services procured;

IPCC _27e_Service Tax_Finance Act 2011 ____________________________ 38

No.1 for CA/CWA & MEC/CEC

MASTER MINDS

b. the recipient of service receives and uses the goods or services, so procured by the service provider in his capacity as pure agent of the recipient of service; c. the recipient of service is liable to make payment to the third party; d. the recipient of service authorises the service provider, to make payment on his behalf; e. the recipient of service knows that, the goods and services, for which payment has been made by the service provider, shall be provided by the third party; f. the payment, made by the service provider on behalf of the recipient of service, has been separately indicated in the invoice, issued by the service provider, to the recipient of the service;

g. the service provider recovers, from the recipient of service, only such amount as has been paid by him to the third party; and h. the goods or services, procured by the service provider, from the third party as a pure agent of the recipient of service, are in addition to the services he provides on his own account. Pure agent: "Pure agent" means a person who a. enters into a contractual agreement with the recipient of service to act as his pure agent to incur expenditure or costs, in the course of providing taxable, service; b. neither intends to hold nor holds any title to the goods or services, so as procured or provided as pure agent of the recipient of service; c. does not use such goods or services so procured; and d. receives only the actual amount incurred to procure such goods or services. Value to include all components even if separately shown: The value of the taxable service is the total amount of consideration consisting of all components of the taxable service and it is immaterial that the details of individual components of the total consideration are indicated separately in the invoice. Inclusions/Exclusions of certain items: Rule 6 of Valuation Rules specifies certain items to be included in or excluded from the value. Q.NO.8. WHAT ARE THE CONDITIONS INCIDENTAL TO THE PAYMENT OF SERVICE TAX? IS THE SERVICE TAX PAYABLE BY THE ASSESSEE, EVEN IN CASES WHERE HIS CLIENTS DO NOT PAY FOR THE SERVICE RENDERED OR PAY ONLY A PART OF THE BILL RAISED IN THIS REGARD? 1. TDS part of Taxable value: Service Tax is to be paid on the value of taxable service, which is charged by a Service Tax assessee. Income Tax, deducted at source, is included in the charged amount. Service Tax is, therefore payable in the total amount, inclusive of the Income Tax, deducted at source. 2. Payable, even if ST is not collected: Failure to realize or failure to charge the Service Tax, at the prevailing rate, does not absolve the liability of the service provider to pay Service Tax. Liability of the service provider to pay Service Tax subsists, even if it is not collected by him from his client.

IPCC _27e_Service Tax_Finance Act 2011 ____________________________ 39

Ph:

0863 22 42 355
Person

www.gntmasterminds.com
Due Date for Payment General: 5th of the month following the quarter of Determination as per POT Rules,2011, i.e. on or before 5th of July, 5th of October and 5th of January Quarter ending in March: Tax should be paid on or before 31st March itself. General: 5th of the month following the month of Determination as per POT Rules, 2011. March: Tax should be paid on or before 31st March itself.

3. Due Date:

a. Individual / Proprietary Firm / Partnership Firm

b. Other Persons

Note: For E-payment of Tax: Due date shall be 6th, instead of 5th. Where the last day is a Public Holiday, payment can be made on the next working day. 4. Form / Challan for Payment: a. Service Tax shall be paid in Form GAR 7 Challan in the Designated Bank Branches. [Note: Payment of Service Tax in Non-Designated Banks does not amount to the payment of Service Tax.] b. A Multiple Service Provider can either use a single GAR 7 Challan for making payment, in respect of all services, or separate challans for each services. 5. Rounding Off [Sec.37D of Central Excise Act]: Amount of Service Tax shall be rounded off to the nearest Rupee. 6. Payment in Cash: Where the amount of Service Tax is paid in Cash, the date of payment is the date, on which cash is tendered to the Designated Bank. 7. Payment in Cheque: a. Date of Deposit = Date of Remittance : Where payment is made in Cheque: It should be deposited with the bank on or before the due date, and If it is realized in the normal course of banking, even after the due date, payment will be deemed to have been made before the due date.

b. Dishonour of Cheque: If the cheque is dishonoured, Service Tax will not be deemed to have been paid.

Illustrations
1. Value of taxable service: Answer the following a. X contracts with Y, a real estate agent, to sell his house and thereupon Y gives an advertisement in television at a cost of Rs. 5 lakhs. Y billed Rs.15 lakhs to X, including charges for television advertisement, showing them separately in the invoice. Mr. Y says that, the value of taxable service in his case is Rs. 10 lakhs only, as he acted as a pure agent of Mr. X, while taking advertisement. Compute the Service Tax to be billed.

IPCC _27e_Service Tax_Finance Act 2011 ____________________________ 40

No.1 for CA/CWA & MEC/CEC

MASTER MINDS

b. In the course of providing a taxable service, a service provider incurs costs such as traveling expenses, postage, telephone, etc., to the extent of Rs. 20,000. He charges Rs. 80,000 for his services and indicates the said costs separately in the invoice issued to the recipient of the service. Compute the amount of Service Tax, to be billed by the service provider. c. A contracts with B, an architect, for building a house and B's fees is fixed at Rs. 3,00,000. During the course of providing the taxable service, B incurs expenses such as telephone charges, air travel tickets, hotel accommodation, etc., totaling Rs. 50,000, to enable him to effectively perform the provision of services to A. Compute the Service Tax liability of B. d. A clearing and forwarding agent charges Rs. 50,000 for his services, which includes octroi charges Rs.10,000, paid on behalf of his client. Compute the value of taxable service and Service Tax liability. e. A cable operator charges Rs. 10,000 for his services, which includes entertainment tax Rs. 2,000, paid on behalf of his client. Compute the value of taxable service and Service Tax liability. Solution: The answers to the aforesaid are as follows: a. Since advertising service is an input service for the estate agent, in order to enable or facilitate him to perform his services as an estate agent, Mr. Y doesn't act as an agent of Mr. X in taking advertisement. Hence, the Service Tax = Rs. 15 lakhs 10.3% = Rs. 1,54,500. b. In this case, the service provider is not acting as an agent of the recipient of service, but procures such inputs or input service, on his own account, for providing taxable service. Such expenses do not become reimbursable expenditure, merely because they are indicated separately in the invoice issued, by service provider to the recipient of service. Hence, the Service Tax = (80,000 + 20,000) 10.3% = Rs.10,300. c. Value of taxable service = 3,00,000 + 50,000 = Rs. 3,50,000, and Service Tax @ 10.3% = Rs. 36,050. d. The clearing and forwarding agent acts as a pure agent of the client while paying octroi charges. Hence, the value of the taxable service = 50,000 - 10,000 = Rs. 40,000, and Service Tax @ 10.3% = Rs. 4,120. e. The cable operator acts as a pure agent of the client, while paying entertainment tax. Hence, the value of the taxable service = 10,000 - 2,000 = Rs. 8,000, and Service Tax @10.3% = Rs. 824. 2. Pure agent: Mr. X, a custom house agent, charges Rs. 1,00,000 from a client. This sum includes Rs. 10,000 towards the payment of customs duty, on behalf of the client. Compute the Service Tax to be charged from the client. Solution: Expenses incurred by the service provider in his capacity as a pure agent do not form the part of the value of taxable service. Here, the custom house agent, Mr. X, has paid Rs. 10,000 towards customs duty on behalf of the client, which is an expenditure incurred by Mr. X in his capacity as the pure agent. Since the sum of Rs.10,000 doesn't include any element of profit, the same will not form a part of the value of taxable service. Hence, the value of taxable service = 1,00,000 10,000 = Rs. 90,000. Service Tax to be charged = 90,000 10.3% = Rs. 9,270.

IPCC _27e_Service Tax_Finance Act 2011 ____________________________ 41

Ph:

0863 22 42 355

www.gntmasterminds.com

3. Pure agent: Mr. Y, an architect, finds a client Mr. A, who need the services of an architect, as well as interior decorator. Mr. A asks Mr. Y to find an interior decorator on his behalf, decide the terms of his engagement and also his remuneration, make payment to him on his (Mr. As) behalf, and provide a consolidated bill, showing relevant break-ups, to him (Mr. A). Mr. Y finds an interior decorator, Mr. Z, whose remuneration is fixed at Rs. 2,00,000 (inclusive of all taxes). Mr. Y charges Rs.8 lakhs towards the value of taxable service provided by him. Compute the amount of the Service Tax to be charged in the bill by Mr. Y if ______ a. The bill amount includes an additional amount of Rs. 2,00,000, towards the cost of interior decorator ; or b. The bill amount includes an additional amount of Rs. 2,50,000, towards the cost of interior decorator. Solution: a. In this case, the bill amount includes only the actual cost of interior decorator. Since Mr. Y has acted as the pure agent of Mr. A, he will get deduction of such cost. Therefore, the value of taxable service provided by Mr. Y = 10,00,000 - 2,00,000 = Rs. 8,00,000, on which the Service Tax = 8,00,000 x 10.3% = Rs. 82,400. CENVAT credit implications: The interior decorator, Mr. Z, will pay Service Tax = Rs. 2,00,000 10.3 110.3 = Rs. 18,676. The client Mr. A can claim CENVAT Credit of Rs. 82,400 + Rs. 18,676 = Rs.1,01,076. b. In this case, bill amount includes amount in excess of the actual cost of interior decorator. Hence, Mr. Y will not be the 'pure agent' of Mr. A, and thus, he will not get any deduction of such cost. Therefore, the value of taxable service provided by Mr. Y = Rs. 8,00,000 + Rs. 2,50,000 = Rs.10,50,000, on which the Service Tax = 10,50,000 x 10.3% = Rs. 1,08,150. CENVAT credit implications: The interior decorator, Mr. Z, will pay the Service Tax = Rs. 2,00,000 x 10.3 110.3 = Rs. 18,676. The services provided by Mr. Z will be treated as input service, in case of Mr. Y and Mr. Y can get CENVAT credit of Rs.18,676, which can be used for the payment of his Service Tax liability. The client, Mr. A, can claim CENVAT credit of Rs. 1,08,150. 4. Gross amount charged and Value of taxable service: Hotel Grand Nagarjuna Palace charges 10% of bill amount as service charges and the Department has asked them to pay Service Tax on it. The assessee has submitted that, the amount @ 10%, collected from the customers, is subsequently disbursed among the staff. Therefore, it is not the part of their income and cannot be included in gross amount charged by them. Examine the case and advise suitably. Solution: As per Sec. 67, the value of taxable service shall be the gross amount charged by service provider for the taxable service provided or to be provided by him. Since the assessee was charging service charges (@ 10% of the bill amount) from its customers for providing the services, the said amount was liable to be included in the gross amount charged. Therefore, service charges (@ 10% of the bill amount) charged by Hotel Grand Nagarjuna Palace, are includible in gross amount charged by them and will be liable to Service Tax.

IPCC _27e_Service Tax_Finance Act 2011 ____________________________ 42

No.1 for CA/CWA & MEC/CEC

MASTER MINDS

5. Determination of Service Tax liability: Y Ltd. a consulting engineer raised a bill of Rs. 2,24,720 (including Service Tax) on his client, for consultancy services rendered by him, in the month of June, 2011. A partial payment of Rs.1,68,450 was received by Y Ltd. in the month of March, 2012. Compute the Service Tax amount payable to Y Ltd. and the due date by which the Service Tax can be deposited. Solution: 1. Principle:

a. Payment on Service provided basis: Though the service provider charges Service Tax in his bill, raised on his client as and when the service is provided, the Service Tax is to be paid to the government only, when the payment is received or invoice issued between 14 days whichever is earlier. Thus, situs of taxation is on service provided date, but the remittance of Service Tax to the government is deferred, till the date of receipt of the taxable service or invoice issued whichever is earlier. b. Due date of Payment of Service Tax: In general, the tax collected should be remitted on or before the 5th of the month following the quarter of collection. In case of the last quarter ending on 31st March, the tax should be paid on or before 31st March itself.
2. Analysis and Conclusion: In the given case, though the service provider has billed a Service charge of Rs.2,03,735 (2,24,720 x 100/110.3).As per Point of Taxation Rules, it could have been paid total service tax of RS.20,985 on or before 5th July,2011.Since the receipt is only Rs.1,68,450, only the proportionate amount of Service Tax of Rs.15,730 (1,68,450 x10.3/110.3) is to be paid and service tax paid in excess of RS.15,730 can be adjusted. 6. Determination of liability - ST not charged separately: Ms. Priya rendered a taxable service to a client. A bill for Rs. 40,000 was raised on 29.4.2011; Rs. 15,000 was received from the client on 1.5.2011 and the balance on 23.5.2011. No Service Tax was separately charged in the bill. The questions are:

a. Is Ms. Priya liable to pay Service Tax, even if the same has not been charged by her? b. In case, she is liable, what is the value of taxable service and the Service Tax payable?
Solution: 1. Service Tax is to be paid, even though the same is not charged separately in the bill.
Date (1) 29.04.2011 01.05.2011 23.05.2011 Nature (2) Bill raised Part Amount Balance Amount Amount received (3) 15000 25000 Taxable service (4) 40000 15000 25000 Service Tax @ 10% (5) 3626 EC @ 3% (6) = (5) x 3% 109 Total Liability (7) 3735 Due date (8) 05.07.2011 -

IPCC _27e_Service Tax_Finance Act 2011 ____________________________ 43

Ph:

0863 22 42 355

www.gntmasterminds.com

7. Liability and Extent of Liability: Ms. Smita rendered taxable services to a client. A bill for Rs.50,000 was raised on 27.04.2011; Rs. 25,000 was received from the client on 04.05.2011 and the balance on 25.05.2011. No Service Tax was separately charged in the bill. The questions are: a. Is Ms. Smita liable to pay Service Tax, even though the same has not been charged by her? b. In case she is liable, what is the value of taxable service and the Service Tax payable? Solution: Case (a): 1. Principles of Law: a. Payable, even if not collected: This liability is not contingent upon the service provider realizing or charging the Service Tax at the prevailing rate. The statutory liability does not get extinguished, if the service provider fails to realize or charge the Service Tax from the service receiver. 2. Conclusion: Hence, Ms. Smita is liable to pay Service Tax. Case (b): 1. Value of taxable service = 2. Analysis & Conclusion: a. The rate of Service Tax payable = 10% + cess 3% = 10.3% b. Value of taxable service Service Tax payable = =

Gross amount charged 100 (100 + Effective Rate)

50,000 100 110.3 50,000 10.3 110.3

= Rs. 45,331 = Rs. 4,669

8. M/s. Renu consultants are a labour contractor of manpower to M/s. Sanu creations. They charge the principal employer, for the wages of their labour, which amounts to Rs.1,20,000, plus their service charge of Rs.12,000 for arranging the labour. The issue is, whether Service Tax is payable on the gross amount charged by them, or only their charges for labour. Examine the case and advise suitably. Solution: a. Taxable Value: Section 67 states that value of any taxable service is the gross amount charged by the service provider, for the taxable services rendered by him. b. Analysis of given case: Value includes recovery of staff costs from the recipient, eg. salary and other contributions. Even if the arrangement does not involve the recipient paying these staff costs to the supplier (because the salary is paid directly to the individual or the contributions are paid to the respective authority), these amounts are still part of the consideration and hence, for part of the gross amount.

IPCC _27e_Service Tax_Finance Act 2011 ____________________________ 44

No.1 for CA/CWA & MEC/CEC

MASTER MINDS

c. Conclusion: Service Tax is to be charged on the full amount of consideration, for the supply of manpower, whether full time or part time. Hence, the taxable amount is the entire amount i.e. Rs. 1,32,000. 9. Determination of Liability of Service Tax: MM Ltd. has entered into a contract with OP Ltd. on 31.05.2011 for rendering services. The contract contains clear details of services. Consideration and Service Tax are charged separately. The following information is also available: a. Advances received in June 2011, from OP Ltd., towards all services = Rs. 1,20,000. b. Total value of services billed to OP Ltd. in August 2011, = Rs. 4,20,000. c. Non-taxable services billed to OP Ltd. [including in (b) above] = Rs. 1,40,000. Balance consideration for services is received in December 2011. 1. How many times does the liability to pay Service Tax arise, in such a case and when? 2. What is the Service Tax liability in each case? 3. What are the due dates for payment in each case? Solution: 1. Liability to pay Service Tax arises only on Mercantile basis. 2. In the given case, liability to pay Service Tax is in respect of a. Receipt of advance money, b. Balance portion of money received, in respect of taxable services.
Towards NonTaxable Service (5) 40,000
1 1,20,000 3

Month

Nature

Amt received/ due (3)

Towards Taxable service (4) 80,000


2 3 1,20,000

Servic e Tax Liabilit y @ 10% (6)

Cess @ 3% (7)

Total Liability @ 10.3% (8)

Due date

(1)

(2)

(9)

June 11

Advance

1,20,000

8,000

240

8,240

5/7/11

August 11

Bill Raised

Though not received, it was due as per PTR (4.2 L1.2L) 3,00,000

2,00,000
2 3

1,00,000

3,00,000

1 3,00,000 3

20,000

600

20,600

5/9/11

Dec. 11

Balance

IPCC _27e_Service Tax_Finance Act 2011 ____________________________ 45

Ph:

0863 22 42 355

www.gntmasterminds.com

10. Ms. Priyanka, a proprietress of Royal Security Agency, received Rs.1,00,000 by an account payee cheque, as advance, while signing a contract for providing taxable service. She received Rs.5,00,000 by credit card while providing the service and another Rs.5,00,000 by a pay order, after completion of the service on 31st January, 2012. All three transactions took place, during the financial year 2011-12. She seeks your advice about her liability towards value of taxable service and the Service Tax payable by her. Solution:
Principle: Gross amount charged means Gross amount charged by a service provider

for the taxable service which is inclusive of Service Tax payable. It includes payment by cheque, credit card, deduction from account and any form of payment, by issue of credit notes or debit notes and book adjustment.
Conclusion: In the given case, Service Tax liability = Rs.11,00,000 x 10.3 / 110.3 = Rs.

1,02,720. 11. Mr. P is a well known interior designer. He has recently designed a 10,000 sq. ft. office of an air conditioner manufacturing company. For this job, he has been paid by the recipient of service, Rs.12,00,000. Besides, the recipient of service has gifted gold ornaments. Service Tax liability (if any) will be met by the recipient of service in addition to the aforesaid payment. Find out the Service Tax liability, under the following two different situations: Situation 1 For a similar job, Mr. P charges at a rate of Rs. 280 per sq.ft. Situation 2 Mr. P has not provided similar service to any other manufacturing company. Equivalent money value of a jewellery is Rs. 9,20,000. The cost of provision of taxable services, provided by Mr. P, in this case is Rs. 22,00,000 Situation 3 In situation 2, assume that cost of provision of taxable services provided by Mr. P in this case is Rs.7,00,000. Solution: Situation Valuation For a similar job, Mr. P charges at a rate of Rs.280 per sq.ft. Therefore, for 10,000 Sq.ft. it is Rs.28,00,000 Amount received in cash Rs. 12,00,000 Monetary value of consideration, in kind, is Rs.9,20,000 Aggregate amount Rs. 21,20,000 However, cost to Mr. P for providing service is Rs. 22,00,000 The aggregate of monetary consideration and consideration in kind is Rs. 21,20,000. Cost to Mr. P for providing service is Rs. 7,00,000. Taxable value of service will be Rs. 21,20,000. Taxable value of service Rs. 28,00,000 Tax liability at 10.30% Rs. 2,88,400

Rs. 22,00,000

Rs. 2,26,600

Rs. 21,20,000

Rs. 2,18,360

IPCC _27e_Service Tax_Finance Act 2011 ____________________________ 46

No.1 for CA/CWA & MEC/CEC

MASTER MINDS

12. An interior decorator charges Rs. 5,50,000 from a client, for providing professional services. The break up of the bill is: Value of furniture sold to the client Labour and facility charges Value of materials consumed in providing the service Compute the amount of Service Tax to be charged from the client. Solution: Rs.2,50,000 Rs.1,50,000 Rs.1,50,000

a. Principles:
Value of furniture sold to client will be exempt under subject to fulfillment of

conditions specified therein.


Labour and facility charges are liable to Service Tax. Value of materials consumed in providing the service forms intrinsic part of the

value service, and is not deductible.

b. Analysis and Conclusion:


Particulars Value of taxable service Service Tax to be charged Computation Rs.1,50,000 + Rs.1,50,000 Rs.10.30% of Rs. 3,00,000 Rs. 3,00,000 30,900

13. Ram Ltd manufactures Turbo Engines for manufacturing plants and also provides installation services. The value of Turbo Engines supplied to its customer including profit element, is Rs.20,00,000. The installation charges towards labour is Rs. 1,00,000, the cost of consumables is Rs.55,000 and the profit element on installation and consumables is Rs. 1,50,000. What is the value of taxable service? Solution: 1. As per Rule 2 of the valuation rules, the value of taxable services does not include the value of material supplied to the workplace. 2. In the given case, value of taxable service is calculated as under: Particulars Cost of labour Cost of consumables Profit Taxable value of services Rs. 1,00,000 55,000 1,50,000 3,05,000

14. Amethyst, Fashion Designers, undertake fashion designing contracts to various events. Services provided to X Ltd. is for Rs. 1,00,000. X Ltd., the manufacturer of cosmetics, gave packs of cosmetics to Amethyst as gift. Find the taxable value of service, if : a. For similar services he charges Rs. 5,00,000. b. Consideration is not known.

IPCC _27e_Service Tax_Finance Act 2011 ____________________________ 47

Ph:

0863 22 42 355
Situation a. Similar services charges are Rs.5,00,000 b. Consideration is not known

www.gntmasterminds.com
Taxable value of service Taxable value of service = Gross amount charged by the service provider to provide similar service, to any other person, in the ordinary course of trade = Rs. 5,00,000 Taxable Value of service will be based on the cost plus reasonable profit. Such cost will have to be worked out, on the basis of usual costing principles, of normal costs and allocation of normal overheads, and including reasonable profit thereon.

Solution:

15. GSA, an excise agent, charges a sum of Rs. 74,000 from his client vide Bill No. 44, dated February 10th, 2012. This sum includes Rs. 45,000 towards the payment of excise duty, on behalf of his client. Compute the Service Tax to be charged from the client. Solution:
Any expenditure incurred by a service provider in his capacity as a pure agent does not

form the part of taxable services.


In the given problem, GSA acts as a pure agent of his client, and Rs. 45,000 is paid to

the excise department, on behalf of the client, which will not be liable for Service Tax.
Consequently, Service Tax to be charged = (Rs. 74,000 Rs. 45,000) x 10.30% = Rs.

2,987. 16. A customer pays only the bill amount, but not the Service Tax amount. How does the service provider work out the Service Tax liability. Should Service Tax be paid, even if not collected from the client or service receiver? Solution:
Principle of Law: Value of taxable services is the Gross amount charged inclusive of

Service Tax. If the Service Tax assessee receives the service charges only, but not the Service Tax payable thereon, the amount so realized from the client would be treated as Gross amount, inclusive of Service Tax.
Example: Assuming on a bill of Rs.1,103 (inclusive of Service Tax of Rs. 103), the service

recipient paid only Rs. 1,000. The Service Tax liability will be worked out as follows: Particulars Value of taxable service (V) Service Tax + Education cess payable Formula (Gross Value x 100) 100 + Rate of tax V x Rate of tax 100 Computation Rs. 1000 x 100 (100 + 10.30) Rs. 907 x 10.30 100 Rs. 907 93 1,000

Total Amount

17. Adlaps Pvt. Ltd. is engaged in providing advertising services. For the month of August 2011, its Gross receipts were Rs. 12,50,000. The break up of these receipts is as follows: Month in which services are performed July August September Receipts (Rs.) 4,30,000 2,50,000 5,70,000

IPCC _27e_Service Tax_Finance Act 2011 ____________________________ 48

No.1 for CA/CWA & MEC/CEC

MASTER MINDS

In the financial year 2010-11 Adlaps Pvt. Ltd. had paid Rs.15,00,000 as Service Tax. State the amount of Service Tax payable by Adlaps Pvt. Ltd. for the month of August 2011, and the date by which it is required to make the payment. Solution:

a. Computation of net Service Tax payable by Adlaps Pvt. Ltd. for August 2011:
Particulars Gross amount charged for the taxable service Service Tax payable at 10.30% Computation Rs.4,30,000 + Rs.2,50,000 + Rs.5,70,000 Rs.12,50,000 x 10.30% Amount Rs.12,50,000 Rs.1,28,750

Note: Sec.67(3) of the Finance Act, 1994 stipulates that the Gross amount charged for the taxable service shall include any amount received towards the taxable service, received before, during or after provision of such service.

b. Due date for the payment of Service Tax: Since the Service Tax liability for the previous year exceeds Rs. 10,00,000, e-payment is mandatory in the current financial year. Therefore, Service Tax has to be paid on or before 6th September, 2011.
18. X & Co., is a service provider. It received Rs.19,80,000 during the financial year 2011-12, after the deduction of tax at source, under Section 194J of the Income-tax Act, 1961. The rate of tax deduction being 10% (i.e. after deduction of Rs. 2,20,000). Calculate the Service Tax liability of X & Co. Can a multiple service provider use a single challan for payment of Service Tax, for various services rendered by it? Solution:

a. Service Tax is to be paid on the value of taxable service, which is charged by a Service Tax Assessee. TDS is included in the charged amount. Service Tax is, therefore, payable on the total amount of inclusive of Income tax, deducted at source.
Particulars Taxable Value (Rs.19,80,000 + Rs. 2,20,000) Service Tax @ 10.3% Rs. 22,00,000 2,26,600

b. Yes. A Multiple Service Provider can either use single GAR 7 Challan for making payment in respect of all services, or separate challans for each services.
19. X Ltd. has agreed to render services to Y. The following are the chronological events: Service Tax Contract for continuing services entered into on August 31st,2011 Advances received in September, 2011, towards all the services Total includes non-taxable services of Balance amount is received in March, 2012 Note: Invoices are issued on the Date of Receipt of consideration itself. Rs. 2,10,000 60,000 70,000

IPCC _27e_Service Tax_Finance Act 2011 ____________________________ 49

Ph:

0863 22 42 355

www.gntmasterminds.com

When does the liability to pay Service Tax arise and for what amount? Contract contains clear details of services; consideration and Service Tax are charged separately, as mutually agreed upon. Solution: In September 2011, Rs. 60,000 is received as advance. The liability to pay Service Tax arises, at the time of receipt of advance or issue of invoice within 14 days whichever is earlier. Further, the liability to pay Service Tax arises at the time of issue of invoice for balance consideration of Rs. 1,50,000 in March, 2012. Moreover, the liability to pay Service Tax arises only upon the receipt of the value of taxable services and not when the bill is raised.
Out of which how much is pertaining to taxable services Rs. 40,000 (Note 1) 1,00,000 (Note 2) Due date Service Tax Education cess Secondary & higher education cess

Month of payment

Amount

Total

If paid electronically

Otherwise

September 2011 March 2012 (Note 3)

60,000

4,000

80

40

4,120

October 6, 2011 October 6, 2011

October 5, 2011 October 5, 2011

1,50,000

10,000

200

100

10,300

Note 1: Rs.60,000 x (Rs. 2,10,000 Rs. 70,000) Rs. 2,10,000 = Rs. 40,000 Note 2: Rs.1,50,000 x (Rs. 2,10,000 Rs. 70,000) Rs. 2,10,000 = Rs. 1,00,000 Note 3: Service Tax is to be paid at the time of issue of invoice or receipt of payment whichever is earlier therefore incase of payment received in March 2012 tax is to be paid during the month ending October 6th /5th. 20. Calculate the net Service Tax payable, under the provisions of Rule 2A of the Service Tax (Determination of Value) Rules, 2006, relating to the determination of the value of services in the execution of a works contract, from the following particulars: a. Gross amount for the works contract (excluding VAT) b. Value of goods and materials sold in the execution of works contract c. CENVAT Credit on (b) above d. Service Tax paid on input services Rs.1,00,000 Rs. 70,000 Rs.1,000 Rs.1,000

e. CENVAT Credit on capital goods issued, in the provision of works contract service Rs.1,000 f. Service Tax rate 10.3%

Make suitable assumptions and provide explanations, wherever required. Solution: Particulars Gross Value charged for works contract Less: Value of Goods and Materials sold Value of Taxable Service Rs. 1,00,000 70,000 30,000

IPCC _27e_Service Tax_Finance Act 2011 ____________________________ 50

No.1 for CA/CWA & MEC/CEC


Service Tax due on the above [10.30 % x 30,000] Less: Service Tax Credit on Input Services

MASTER MINDS
3,090 (1,000) 2,090 (1,000) 1,090

Less: CENVAT on Capital Goods issued in provision of Works Contract Service Net Service Tax Payable

Note: Since goods not included in the value of taxable service, CENVAT credit on the goods cannot be availed. 21. For certain taxable services rendered by P, VAT as well as Service Tax is leviable. The following bill was raised by P on V (Service Receiver) on 20.03.2012: Amount of Bill VAT Total Rs. 40,000 Rs. 400 Rs. 40,400

On 31.3.2012, P receives Rs. 30,000 in full settlement. What is the Service Tax payable? You are informed that V has incurred hotel bills of Rs.3,708 on behalf of P. Indicate the provisions considered in arriving at the Service Tax payable. Solution:

1. Value of Taxable Service is Rs. 30,000 plus consideration received in kind as per the Valuation Rules. 2. Service Tax Payable = [(Rs. 30,000 + Rs. 3,708) = Rs. 33,708 x 10.30 / 110.30] = Rs. 3,148
22. Prahlaad has paid the amount of Service Tax for the quarter ending 30th June, 2011, by cheque. The date of presentation of cheque to the designated bank is 5th July, 2011, and it is realized by the bank on 7th July, 2011. What is the date of payment of Service Tax in this case? Whether any interest and penalty is attracted in this case? Solution: 1. Payment in Cheque: a. Date of Deposit = Date of Remittance : Where payment is made in Cheque: It should be deposited with the bank on or before the due date, and If it is realized in the normal course of banking, even after the due date, payment will be deemed to have been made before the due date. b. Dishonour of Cheque: If the cheque is dishonoured, Service Tax will not be deemed to have been paid. 2. Due Date: In case of individual 5th of the month following the quarter of collection, i.e. on or before 5th of July, 5th of October, 5th January and in case of March, 31st March itself. 3. Conclusion: The date of payment of Service Tax is 5th July 2011, i.e. on the due date itself. Therefore no interest and penalty are attracted in this case.

IPCC _27e_Service Tax_Finance Act 2011 ____________________________ 51

Ph:

0863 22 42 355

www.gntmasterminds.com

23. R Ltd. gives the following particulars relating to the service provided by it to its various clients for the month ending 31.03.2012: i. Total bills raised for Rs. 17,50,000 out of which bill for Rs.1,50,000 was raised on an approved International Organization and payments for bills for Rs. 2,00,000 were not received until 31.03.2012

ii. Amount of Rs. 1,00,000 was received as an advance from XYZ Ltd. on 25.03.2012 to whom the services were provided in April 2012. Compute: a. Value of Taxable Services. b. Amount of Service Tax Payable. c. Last date of Service Tax payable. Solution:

a. Value of Taxable Services for the month ending 31.03.2012


SL No. 1. 2. Particulars Total bills raised during the month ending 31.03.2012 Less: Bills raised on an approved International Organization due to Exemption granted vide Notification No. 16/2002 dated 02.08.2002 Bills in respect of which payment was not received until 31.03.2012 No adjustment is required because according to Rule 3 of point of Taxation Rules, 2011 where invoice is issued within 14 days from the date of completion of the service, the point of Taxation will be earlier of the following two dates: 3. a. Issue of Invoice for the service provided ; or b. Date of payment to the extent it is received In the absence of my specific information in the question, it is assumed that invoice has been issued within 14 days from the date of completion of the taxable service. Consequently, service Tax is required to be paid on the basis of issue of invoice even if no payment has been received until 31.03.2012 Add: Amount of advance received on 25.03.2012 is to be included in the taxable value of service because according to Explanation appended to rule 3 of point of Taxation rules, 2011 wherever any advance by whatever name known is received by the service provider towards the provision of taxable service, the point of taxation shall be the date of receipt of each such advance. Value of Taxable Service Amount (Rs.) 17,50,000 1,50,000

4.

1,00,000

17,00,0000

b. Amount of service tax payable.


S.No. 1. 2. Particulars Value of Taxable Service Service Tax @ 10.30% Amount 17,00,000 1,58,749

IPCC _27e_Service Tax_Finance Act 2011 ____________________________ 52

No.1 for CA/CWA & MEC/CEC

MASTER MINDS

c. Last date of service tax payable: Since the Service Tax is required to be paid for the month of March 2012, last date for making payment of Service Tax payable will be 31.03.2012 in accordance with second proviso to rule 6(1) of Service Tax Rules, 1994.
24. G, a Practicing Chartered Accountants provides the following particulars, for the Quarter ending 31.12.2011. All bills raised by G are Inclusive of Service Tax, if applicable. Date 25.09.2011 05.10.2011 19.10.2011 22.10.2011 28.10.2011 04.11.2011 04.01.2012 Particulars Bill no.1010 in r/o Statutory Audit Fee charged to R. Ltd. Bill no.1011 in r/o Internal Audit Fee charged to S. Ltd Amount Received from R Ltd. In respect of Bill No.1010 Bill no. 1012 in r/o Appearance before Appellate Tribunal against notice issued to T Ltd. Amount received from T Ltd. in r/o Bill No.1012 Amount received from S Ltd against bill no. 1011 Balance Amount Received from S Ltd. against Bill No .1011 Amount 3,00,000 2,40,000 3,00,000 2,60,000 2,60,000 1,60,000 80,000

You are required to compute the following for the quarter ending 31.12.2011 a. Value of Taxable Service b. Amount of Service Tax payable c. Last date of Service Tax payable Solution:

a. Value of Taxable Service rendered by G as a practicing Chartered Accountant for the Quarter Ending 31.12.2011: Since G is engaged in providing practicing Chartered Accountants Services, he will fall within the ambit of Rule 7(c) point of Taxation Rules, 2011. Accordingly, in the case point of Taxation will be the date of which Payment is received. In simple words, the basis for discharging Service Tax liability in case of Practicing Chartered Accountants Services has not under gone any change despite the fact that point of Taxation Rules, 2011 have compulsorily come in to force w.e.f. 01.07.2011. In addition, it is worth highlighting that service in respect of representing a client before a statutory authority have become taxable w.e.f. 01.05.2011 by virtue of Notification No.32/2011 dated 25.04.2011. As a result, all service provided or to be provided by a practicing Chartered Accountant have become taxable w.e.f. 01.05.2011.
In view of above legal position, the value of services during the quarter ending 31.12.2011 has been computed as under: Value of Taxable Service during the quarter ending 31.12.2011 S.No. 1. 2. 3. Date 19.10.2011 28.10.2011 04.11.2011 Particulars Amount Received from R Ltd. in respect of Bill No.1010 Amount received from T Ltd. in r/o Bill No.1012 Amount received From S Ltd. against Bill No.1011 Total taxable value of Service Value of Taxable Services 2,71,985[3,00,000 X 100/110.30] 2,35,721[2,60,000 X 100/110.30] 1,45,059[1,60,000 X 100/110.30] 6,52,765

IPCC _27e_Service Tax_Finance Act 2011 ____________________________ 53

Ph:

0863 22 42 355

www.gntmasterminds.com

a. Since amount of bills are inclusive of Service Tax, value of Taxable Services has been determined by working backwards i.e. Amount of Bill X 100/110.30 b. Since balance amount of Rs.80, 000 (inclusive of Service Tax) has not been received from S Ltd. during the quarter ending 31.12.2011, the same has not been taken into consideration for the purpose of working out value of taxable Service for the quarter ending 31.12.2011. However, the aforementioned amount of Rs.80, 000 under the heading Taxable Amount Charged in ST Return.

b. Amount of Service Tax payable for the quarter ending 31.12.2011


S. No. 1. 2. Particulars Value of Taxable Service Service Tax @ 10.30% Amount 6,52,765 67,235

c. Last date of service tax payable: Since the Service Tax is required to be paid for the quarter ending 31.12.2011, last date for making payment of Service Tax will be as under [depending upon the mode of making payment of Service Tax] in accordance with first proviso to rule 6(1) of Service Tax Rule, 1994.
Mode of making payment of Service Tax Electronically through internet banking Any mode other than e-banking such as cash/ Cheque / Credit card etc. Last Date for Depositing Service Tax for the quarter ending 31.12.2011 06.01.2012 05.01.2012

25. R Ltd.[a non-small scale sector assesses] submitted the following information: Service Tax liability for the month of June 2011 Due date of discharging above liability Date on which above Service Tax liability was actually discharged Rs.4,50,000 05.07.2011 14.12.2011

Compute the amount of interest to be paid by R Ltd. for delay in payment of Service Tax under Section 75 Solution: The amount of interest has been computed in the following manner. S.No 1. 2. 3. 4. 5. 6. Particulars Relevant Amount of Service Tax Due date of discharging above liability Date on which above Service tax liability was actually discharged Delay in terms of No. of days Rate of Interest Amount of Interest[1 X 4 X 5] Details Rs.4,50,000 05.07.2011 14.12.2011 162 18% p.a [Since R Ltd is a nonsmall scale sector assessee] Rs.35,951

IPCC _27e_Service Tax_Finance Act 2011 ____________________________ 54

No.1 for CA/CWA & MEC/CEC

MASTER MINDS

26. Kirti Ltd. has entered into contract with H Ltd. on 05.07.2011 for rendering both taxable and non-taxable services for an aggregate amount of Rs.31, 50, 000 consisting of Rs.21, 00, 000 for taxable services and Rs. 10, 50, 000 for non-taxable services. Following further information is given in this respect: i. Advance of Rs. 9,00,000 [exclusive of service Tax] received on 05.07.2011 from H Ltd. towards both towards taxable and non-taxable Services

ii. The entire contract was completed on 24.08.2011 and invoice was issued on 02.09.2011 for above mentioned contract amount of Rs.31,50,000 iii. Balance payment of Rs.22,50,000[Rs 31,50,000 Rs 9,00,000] was, however, received on 19.03.2012 From the above details, please determine the following: a. Point of Taxation as per point of Taxation Rules, 2011. b. Amount of Service Tax payable. c. Last date for making payment of Service Tax without any interest. d. Will there be any change in point of Taxation and last date for making payment of Service Tax without any interest if invoice is issued on 10.09.2011 instead of 02.09.2011. Solution (i) Determination of Point of Taxation. Particulars Point of Taxation Remarks Explanation to Rule 3 of point of Taxation Rules, 2011 provides that wherever any advance by what ever name known is received by the service provider towards the value of provision of taxable service, the point of Taxation shall be the date of receipt of each such advance. According to Rule 3 of point of Taxation Rules.2011 when the invoice is issued within 14 days from the date of completion of the service, point of Taxation will be earlier of the following two dates: i. 02.09.2011 Issue of invoice for the service provided or to be provider. Or

Receipt of advance of Rs.9,00,000 [exclusive of Service Tax] on 05.07.2011

05.07.2011

Issue of invoice for Rs.31,50,000 on 02.09.2011 consequent upon completion of contract on 24.08.2011

ii. Date of payment to the extent it is received. Since in the present case invoice has been issued within 14 days from the date of completion of invoice, point of Taxation shall be date of issue of invoice i.e.2.09.2011 Rs.22,50,000 forms part of total contract of Rs.31,50,000. Point of Taxation in r/o Rs.31,50,000 has already been determined above in two stages i.e. on receipt basis in r/o advance and on issue of invoice basis in r/o remaining Rs.22,50,000.Thus when outstanding payment of Rs.22,50,000 is received, there will be no point of Taxation.

Receipt of balance amount of Rs.22,50,000 on 19.03.2012 after adjusting advance of Rs.9,00,000

IPCC _27e_Service Tax_Finance Act 2011 ____________________________ 55

Ph:

0863 22 42 355

www.gntmasterminds.com

(ii) Amount of Service Tax payable: Since Taxable Services and Non-Taxable are to be provided for Rs.21,000 for Rs.10,50,000 respectively, the ratio of Taxable to Non-Taxable Services works out to be 2:1 Accordingly, the amount of Service tax payable has been calculated as under: Total Amount Rs. 9,00,000 Rs.22,50,000 [out of total of Rs.31,50,000, above mentioned advance of Rs.9,00,000 is to be deducted] Division of Total Amount into Taxable and Non-Taxable Portions Taxable Rs.6,00,000 Non- Taxable Rs. 3,00,000 Taxable Rs.15,00,000 Non-Taxable Rs.7,50,000 Service Tax Payable on Taxable portion @ 10.30% Rs.61,800 [Rs.6,00,000 X 10.30%] Rs.1,54,000 [Rs.15,00,000 X 10.30%]

(iii) Last date for making payment of Service Tax without any Interest Taxable Amount Rs. 6,00, 000 Point of Taxation 05.07.2011 Last date for making payment of Service Tax with out any interest 05.08.2011/06.08.2011 depending upon nonelectronic / electronic mode of making payment respectively. 05.10.2011/06.10.2011 depending upon nonelectronic / electronic mode of making payment respectively

Rs.15,00,000

02.09.2011

(D) Consequence of issuing invoice on 10.09.2011 According to Rule 3 of point of Taxation Rules. 2011 where invoice is not issued within 14 days from the date of completion of her service, point of Taxation shall be earlier of the following two dates: Point of Taxation 24.08.2011 i. Date of Completion of Service ; or ii. Date of payment of Service to the extent it is received

Accordingly in the present case the date of completion of service will be point of Taxation. Last date for 05.09.2011/06.09.2011 As point of Taxation is 24.08.2011, last making depending upon non- date for making payment of service Tax payment of electronic/electronic mode has been computed accordingly. service Tax of making payment of service Tax. In other words, there is no change in the last date for making payment of Service Tax. Note: Issue of invoice on 10.09.2011 has no impact on point of Taxation as well as Last date for making payment of Service Tax in respect of Taxable portion of Rs.6, 00, 000 out of Total Advance of Rs.9,00,000.

IPCC _27e_Service Tax_Finance Act 2011 ____________________________ 56

No.1 for CA/CWA & MEC/CEC

MASTER MINDS

27. Raj Ltd. provides the following information for the half year ending 30.09.2011 i. Total bills raised (exclusive of Service Tax) Rs. 68,00,000 which interalia include the following. a. Service provided for the official use of a foreign Diplomatic Mission in India=Rs. 6,00,000 b. Service rendered to a Developer of Special Economics Zone which are wholly Consumed with in SEZ for authorized operations = Rs.4,00,000 ii. Amount of Rs.30, 00, 000 (including Rs. 6, 00, 000 for service rendered to a Diplomatic mission) was received on 18.08.2011. These services were provided on 16.07.2011 and invoice was also raised on the same date. iii. Amount of Rs.32, 00,000 (including Rs.4, 00, 000 for wholly consumed service within SEZ) was received on 22.09.2011. These services were provided on 25.08.2011 & invoice was raised on 29.08.2011 iv. Advance of Rs.6,00,000 [exclusive of Service Tax] was also received on 10.09.2011 against the services which will be provided on 11.10.2011 From the above details, determine the following:

a. Point of Taxation b. Value of Taxable Services and Amount of Service Tax Payable c. Last Date for depositing Service Tax without any interest.
Solution: A. Determination of point Taxation Particulars Issue of invoice for Rs.24,00,000 on 16.07.2011 consequent upon completion of Service on 16.07.2011. It is also worth highlighting that provision of Service to foreign Diplomatic Mission for official use in India has been specifically exempted from levy of Service Tax by virtue of Notification No. 33/2007 dated 23.05.2007 Point of Taxation Remarks According to Rule 3 of point of Taxation Rules, 2011 when the invoice is issued with in 14 days from the date of completion of the service, point of Taxation will be earlier of the following two dates. i. 16.07.2011 Issue of invoice for the service provided or to be provided; or

ii. Date of payment to the extent it is received Since in the present case in voice has been issued within 14 days from the date of completion of service, point of Taxation shall be date of issue of invoice i.e.,16.07.2011. Provisions of above-mentioned Rule 3 will apply in the present case also. Since in the present case also invoice has been issued within 14 days from the date of completion of service [25.08.2011], point of Taxation shall be Date of issue of invoice i.e. 29.08.2011

Issue of invoice for Rs.28,00,000 on 29.08.2011 consequent upon completion of service on 25.08.2011. It is also worth highlighting that provision of Services[which are wholly consumed in SEZ] to Developer

29.08.2011

IPCC _27e_Service Tax_Finance Act 2011 ____________________________ 57

Ph:

0863 22 42 355

www.gntmasterminds.com

of Special Economic Zone has been specifically exempted from levy of service Tax by virtue of Notification No. 17/2011 dated 01.03.2011 Receipt of advance amount of Rs. 6,00,000 on 10.09.2011 against taxable services to be provided on 11.10.2011 10.09.2011

Explanation to Rule 3 of point of Taxation Rules, 2011 provides that wherever any advance by whatever name known is received by the service provider towards the value of provision of taxable service, the point of Taxation shall be the Date of receipt of each such advance. Since in the present case advance has been received on 10.09.2011, point of Taxation shall be receipt of such advance i.e.10.09.2011

(B) Value of Taxable Services & Amount of Services Tax Payable Value of Taxable Services Rs.24,00,000 Point of Taxation 16.07.2011 Amount of Service Tax Payable Rs.2,47,200 Remarks Specific Exemption to service provided to foreign Diplomatic Mission in India as mentioned above Specific Exemption to service provided to Developer of Special Economic Zone as mentioned above Taxable in accordance with Explanation to Rule 3 of point of Taxation Rules, 2011 as mentioned above.

Rs.28,00,000

29.08.2011

Rs.2,88,400

Rs.6,00,000

10.09.2011

Rs.61,800

(c) Last date for depositing Service Tax without any Interest Taxable Amount Rs.24,00,000 Point of Taxation 16.07.2011 Last date for making payments of service Tax without any interest 05.08.2011/06.08.2011 depending upon nonelectronic/electronic mode of making payment respectively 05.09.2011/06.09.2011 depending upon nonelectronic/electronic mode of making payment respectively. 05.10.2011/06.10.2011 depending upon nonelectronic/electronic mode of making payment respectively.

Rs.28,00,000

29.08.2011

Rs. 6,00,000

10.09.2011

IPCC _27e_Service Tax_Finance Act 2011 ____________________________ 58

No.1 for CA/CWA & MEC/CEC Self Test Questions

MASTER MINDS

1. Write a brief note explaining the circumstances, under which the expenditure or costs incurred by a service provider, as a pure agent of the recipient of service, shall be excluded from the value of taxable services, under the Service Tax (Determination of Value) Rules, 2005. 2. Whether the initial deposit made by a subscriber, at the time of application for a telephone connection, is includable in the taxable value? 3. Service Tax (Determination of Value) Rules, 2006 (Valuation Rules) and Taxation of Services (Provided from outside India and received in India) Rules, 2006 (Import Rules) came into force from 19.4.2006 onwards. Answer the following, with reference to the said Rules:

a. Expenditure or costs incurred by the service provider, as a Pure agent of the recipient of service, shall be excluded from the value of the taxable service. Who is a pure agent, under the valuation rules? b. What is the value of taxable service, in the case of service provided from outside India, under the valuation rules? c. What are the three categories specified in Rule 3 of the Import Rules?

IPCC _27e_Service Tax_Finance Act 2011 ____________________________ 59

Ph:

0863 22 42 355
QUESTION Briefly explain the export of services Briefly explain the import of services

www.gntmasterminds.com
ABC A A C

PART 4. EXPORT & IMPORT OF SERVICES & REFUNDS


NO. 1. 2. 3.

Write short notes on rebate, in respect of Service Tax paid on exports vide notification, no.11/2005 st dt. 19-04-2005.

Q.NO.1 BRIEFLY EXPLAIN THE EXPORT OF SERVICES Criteria for decision of Exports, based on the location of immovable property: 1. Export [Rule 3(1)(i) of Export Rules, 2005]: Taxable service will be considered as export, if: a. Nature of Property: Such services are in relation to an immovable property. b. Location of Property: The immovable property is situated outside India. 2. Other Conditions [Rule 3(2) of Export Rules, 2005]: a. Service is provided from India and used outside India, and b. Payment is received in Convertible Foreign Exchange. Criteria based on the performance of service: 1. Export [Rule 3(1)(ii) of Export Rules, 2005]: Taxable service will be considered as export, if such a service is performed outside India. Even if a part of the service is performed outside India, it will be treated as export. 2. Other Conditions [Rule 3(2) of Export Rules, 2005]: a. Service is provided from India and used outside India, and b. Payment is received in Convertible Foreign Exchange. Criteria based on the location of recipient: 1. Recipients: For the purpose of treating a taxable service as export, based on the location of recipients, the recipients are classified as follows: a. Industrial /Commercial Concerns Having an office or commercial establishment in India. Not having office or commercial establishment in India. b. Others (i.e. other than Industrial/Commercial Concerns) 2. Export [Rule 3(1) (iii) (c) of Export Rules, 2005]: Taxable service will be treated as export, as follows based on nature of the recipient:

IPCC _27e_Service Tax_Finance Act 2011 ____________________________ 60

No.1 for CA/CWA & MEC/CEC


Industrial/Commercial Concerns Having an office or Not having office commercial or commercial establishment in establishment in India India Should be located Should be located outside India. outside India. From the Concerns Office/Establishment outside India.

MASTER MINDS
Others (i.e. other than Industrial/Commercial Concerns) Should be located outside India, at the time of receipt of service. From the recipient.

Recipient Order for Service

From the recipient.

3. Other Conditions [Rule 3(2) of Export Rules, 2005]: a. Service is provided from India and used outside India, and b. Payment is received in Convertible Foreign Exchange. Q.NO.2 BRIEFLY EXPLAIN ABOUT IMPORT OF SERVICES. Criteria for decision of Imports based on the location of immovable property: 1. Import [Rule 3(i) of Taxation of Service (Provided from outside India and received in India) Rules, 2006): Taxable service will be considered as import, if a. Nature of Property: Such services are in relation to an immovable property. b. Location of Property: The immovable property is situated in India. 2. Other Conditions (Section 66A): a. Service Provider: General: Service provider has established a business or a fixed place of business outside India; Located in more than one country: Where the service provider has business establishments in more than one country, the establishment directly connected with the provision of service shall be treated as the place, from where the service is provided. b. Service Receiver: Service receiver has his place of business, fixed establishment, Permanent Address or usual place of residence, in India. Criteria based on the performance of service: 1. Import [Rule 3(ii) of Taxation of Services (Provided from outside India and received in India) Rules, 2006]: Taxable service will be considered as import, if such a service is rendered, wholly or partly in India. 2. Other Conditions [Section 66A]: a. Service Provider: General: Service provider has established a business or a fixed place of business outside India;

IPCC _27e_Service Tax_Finance Act 2011 ____________________________ 61

Ph:

0863 22 42 355

www.gntmasterminds.com

Located in more than one country: Where the service provider has business establishments in more than one country, the establishment directly connected with the provision of service shall be treated as the place, from where the service is provided. b. Service Receiver: Service receiver has his place of business, fixed establishment, Permanent Address or usual place of residence, in India. Criteria based on the location of recipient: 1. Import: Taxable service shall be treated as import, if it is received by a recipient, located in India, for use in relation to business or commerce. 2. Other Conditions [Section 66A]: a. Service Provider: General: Service provider has established a business or a fixed place of business outside India; Located in more than one country: Where the service provider has business establishments in more than one country, the establishment directly connected with the provision of service shall be treated as the place, from where the service is provided. b. Service Receiver: Service receiver has his place of business, fixed establishment, Permanent Address or usual place of residence, in India. c. Usage: Service received by the recipient is in relation to his business or commerce. Q.NO.3. WRITE SHORT NOTES ON REBATE, IN RESPECT OF SERVICE TAX PAID ON EXPORTS, VIDE NOTIFICATION No.11/2005 ST Dt. 19-04-2005. 1. Eligible Exports: Exports made to any country, other than Nepal and Bhutan. 2. Rebatable Amount: Whole of Service Tax and Cess, paid on all taxable services exported. 3. Conditions: a. Export: Services are exported in terms of Rule 3 of Export of Services Rules, 2005. b. Convertible Foreign Exchange: Payment for the taxable services has been received in India in Convertible Foreign Exchange. c. Payment of Tax: Service Tax and Cess (EC and SHEC) should have been paid, on the taxable services exported. d. Minimum Rebate: Minimum amount of Rebate of Duty, Service Tax and Cess admissible must be Rs.500 or more. 4. Procedure for claiming rebate: Aspect Application for rebate Description After the services are exported, the claim of rebate should be filed, in Form ASTR-1, to the jurisdictional AC/DC.

IPCC _27e_Service Tax_Finance Act 2011 ____________________________ 62

No.1 for CA/CWA & MEC/CEC


The application should be enclosed with:

MASTER MINDS

a. Proof of Payment: Evidence for payment of Service Tax/cess, on taxable services exported, for which the rebate is claimed. Enclosures to application b. Proof of Receipt: Evidence of receipt of payment, against taxable services exported. c. Declaration: Declaration, that the taxable service has been exported in terms of Rule 3 d. Evidence of Export: Documents evidencing the export of taxable services. Rebate AC/DC shall sanction the rebate, either in whole or part, if he is satisfied that the claim is in order.

5. Recovery of rebate: Rebate paid shall be recovered, with interest, u/s 73 and 75 under the following circumstances a. Service Taxes and Cess, rebate of which have been claimed, have not been paid by the service provider, b. Taxable service, rebate for which have been claimed, have not been exported.

Illustrations
1. X Ltd., an Indian company, has one more place of business in the United States of America. Y Ltd., another Indian Company exported services to a unit of X Ltd., which is situated in the USA. What is the legal position relating to Service Tax?

a. If the order for services rendered from the US office of X Ltd.? b. If the order for services not rendered from the US office of X Ltd.?
Solution: 1. In both the cases (a) and (b), services are rendered outside India, Hence, the services are not taxable in India. 2. Irrespective of the office from which order is rendered, they are considered as export of services.

IPCC _27e_Service Tax_Finance Act 2011 ____________________________ 63

Ph:

0863 22 42 355
QUESTION

www.gntmasterminds.com
ABC A C C C

PART 5. GENERAL EXEMPTION FROM SERVICE TAX & SSP


NO. 1. 2. 3. 4. Explain the conditions to be satisfied, for availing the basic exemption limit of Rs.10 lakhs (SSP). Explain exemption relating to the services, provided by public authorities, while performing statutory functional duties. What are the alternate rates, of tax in Service Tax, as outlined in the Service Tax Rules? What are the general exemptions, which are applicable to all services?

Q.NO.1.EXPLAIN THE CONDITIONS TO BE SATISFIED FOR AVAILING THE BASIC EXEMPTION LIMIT OF Rs.10 LAKHS (SSP). a. Exemption: The aggregate value of taxable services not exceeding Rs.10 Lakhs (i.e. the sum total of the first consecutive payments), received by the service provider during a financial year, is exempt from tax. The service provider will be liable to pay tax, only in respect of amount Received, in excess of Rs.10 Lakhs. b. Turnover Limit: This exemption is applicable for small service providers, whose turnover (aggregate value of taxable service rendered), from one or more premises, did not exceed Rs.10 lakhs, during the preceding financial year. The aggregate value is determined, considering together, the service provided from all the premises of the service provider. Note: Aggregate value of services, provided by an assessee includes all types / classification of services. c. Nature of Exemption: i. The exemption benefit is optional, and the service provider can choose not to opt for this exemption.

ii. However, the option once exercised, shall not be withdrawn, during the remaining part of the financial year. d. Conditions as to CENVAT Credit: i. Service provider shall not avail the CENVAT Credit of: Service Tax paid on any input services, used for providing the taxable service, for the exemption limit. Duty paid on capital goods received, during the period of exemption. ii. Eligible Credit: The service provider can claim CENVAT credit, only in respect of inputs/input services received after the exemption period (i.e., after the receipts exceeds Rs.10 lakhs, during the financial year). Where capital goods have been received during the previous financial year, the service provider can claim the balance credit in respect of such capital goods (i.e. balance of 50% of CENVAT Credit).

IPCC _27e_Service Tax_Finance Act 2011 ____________________________ 64

No.1 for CA/CWA & MEC/CEC


e. No Basic Exemption for the following persons: i.

MASTER MINDS

Brand / Trade Name Users: Persons using Brand Name / Trade Name of another person cannot avail benefit of threshold exemption in respect of the services provided, under such Brand / Trade name.

ii. Service receivers Paying Tax: Service receivers, who are liable to pay Service Tax in terms of Rule 2(1)(d) cannot avail the benefit of threshold exemption, i.e. Importers of Services, Specified Consignors / Consignees in respect of Goods Transport Agency, etc. Q.NO.2. EXPLAIN EXEMPTION RELATING TO SERVICES, PROVIDED BY PUBLIC AUTHORITIES, WHILE PERFORMING STATUTORY FUNCTIONAL DUTIES. a. Statutory activities: Service Tax shall not be leviable on fee collected by Public authorities, while performing statutory functions / duties, under the provisions of a law. If the authority performs a service, which is not in the nature of statutory activity, and the same is undertaken for a consideration not in the nature of statutory fee / levy, then in such cases, Service Tax would be leviable. b. Reason for not levying Service Tax: i. The activities performed by the Sovereign/ public authorities, under the provision of law, are in the nature of statutory obligations, which are to be fulfilled in accordance with the law.

ii. The fee collected by them, for performing such activities, is in the nature of compulsory levy, as per the provisions of the relevant statute, and it is deposited into Government Treasury. iii. It is in the public interest and undertaken as a mandatory and statutory function. iv. The activities are not in the nature of service to any particular individual, for consideration. c. Examples: i. Verification, approval and calibration of weighing and measuring instruments, undertaken by Regional Reference Standards Laboratories (RRSI),

ii. Fitness Certificate, issued to vehicles, by Regional Transport Officer (RTO), iii. Inspection and Certificate for Boilers, issued by Directorate of Boilers, iv. Inspection and Certificates, issued for petroleum storage tank, LPG / CNG storage tank by Explosive Department. Q.NO.3. WHAT ARE THE ALTERNATE RATES OF TAX, IN SERVICE TAX, AS OUTLINED IN THE SERVICE TAX RULES? a. Air Travel Agent: i. Applicability: Air Travel Agent has the option to pay Service Tax (monthly), as stated under, instead of paying Service Tax at the normal rates.

ii. Basic Fare: The commission is payable by the Airlines, on this part of fare.

IPCC _27e_Service Tax_Finance Act 2011 ____________________________ 65

Ph:

0863 22 42 355

www.gntmasterminds.com

iii. Rate: 0.6% of Basic Fare, in case of Domestic Bookings, and 1.2% of Basic Fare, in case of International Bookings. EC @ 3% on ST is payable. b. Life Insurance Business: i. Applicability: Premium in respect of Composite Policies i.e. Risk & Savings, 100% Risk Cover Policy etc., is not eligible for alternate rate of tax.

ii. Conditions: Break up of the premium amount is not disclosed, in any of the document, given to the policy holder. iii. Rate: Service Tax can be paid @ 1.5% of Premium, on such composite policies. c. Works contract: i. Applicability: A person is liable to pay Service Tax, in relation to works contract services. The service provider should exercise the option to pay Service Tax, under composition scheme, before the commencement of works contract. Option once exercised cannot be withdrawn, till the end of works contract. (exercise of option is contract wise & not year-wise) iii. Rate: Service Tax can be paid @ 4% of the Gross amount charged (excluding VAT / Sales Tax, paid by the works contractor) d. Banking & Financial Services: i. Applicability: Person: Foreign Exchange Broker, Authorised Dealer in Foreign Exchange, Authorised Money Changer. Activity: Services in relation to purchase or sale of Foreign Currency, including money changing. ii. Condition: Consideration is not shown separately in the invoice, bill or challan. iii. Rate: Service Tax can be paid at the rate of 0.25% of Gross amount of Currency Exchanged. Q.NO.4. WHAT ARE THE GENERAL EXEMPTIONS, WHICH ARE APPLICABLE TO ALL SERVICES? The general exemptions, which are applicable to all services, are summarized as below: Services provided to United Nations, or an International Organization. Services provided to Developer/Units in Special Economic Zones. Exemption for goods and materials, sold by the service provider.

ii. Conditions:

IPCC _27e_Service Tax_Finance Act 2011 ____________________________ 66

No.1 for CA/CWA & MEC/CEC Illustrations

MASTER MINDS

1. ABC & Co. receives the following amounts during the half year ending March 31st, 2012: Particulars a. For services performed prior to the date of levy of Service Tax (assume Service Tax was levied from a specified date by change of law) b. Advance amount received in March, 2012 (no service is rendered and the amount is refunded to the client, in July, 2012) c. For free services rendered to customers, amount reimbursed by the manufacturer of such product (for the period after the imposition of service tax) d. Amounts realized and, on which, Service Tax is payable (excluding the items above) Calculate the Service Tax liability duly considering the threshold limit. Solution: Computation of Service Tax liability of ABC & Co Advance received in March, 2012 (it is assumed that, a bill was issued to get advance) (a) Amount reimbursed by the manufacturer (b) Other amount realized [it is assumed that, bills were issued before issuing the bill mentioned in (a)] (c) Total [(a)+(b)+(c)] Less: Threshold limit (it is assumed that ABC & Co. has not provided any taxable service in any preceding year) Balance Service Tax (Rs. 5,51,500 x 10.3 / 110.3) Rs. 75,000 50,000 14,26,500 15,51,500 10,00,000 5,51,500 51,500 Amount 3,50,000 75,000 50,000 14,26,500

Note Against the bill issued [which is mentioned in (a)], no service is provided. If the amount is refunded, along with Service Tax to the service receiver, the excess Service Tax can be adjusted. 2. On April 1st, 2011, X is an unregistered service provider. He provides the following details in respect of taxable services, provided during the financial year, 2011-12. Bill. No 100 Date of transaction June 30, 2011 July 14, 2011 Amount of bill (excluding service tax) Rs. Payment received Rs. 1,00,000

Particulars

Advances received from a customer (on July 14, 2011 Bill. No 100 was issued) Bill.No.100 issued

100

1,00,000

IPCC _27e_Service Tax_Finance Act 2011 ____________________________ 67

Ph:

0863 22 42 355
Sep 30, 2011 Sep 30, 2011 Dec 31, 2011 Bill. No. 101 issued

www.gntmasterminds.com
9,50,000 3,00,000 5,00,000 -

101 101 102

Part payment received against Bill No.101 Bill.No.102 issued (for taxable services, rendered during December, 2011) Money received against Bill.No.102 Bill.No103 issued (for taxable services rendered during January, 2012) Money received against Bill.no.103 Bil.No.104 issued (for taxable services rendered during March, 2012) Money received against Bill.No.104

102 103

Dec 31, 2011 Jan 31, 2012

1,00,000

3,00,000 -

103 104

Jan 31, 2012 March 31, 2012 March 31, 2012

2,00,000

1,00,000 -

104

2,00,000

* Service Tax is collected extra wherever it is applicable. The Service Tax provider complies with the provisions of registration and collection of Service Tax as per Service Tax laws. He gets registered during the financial year. Compute the Service Tax liability of the service provider for the year 2011-12, considering the rate of Service Tax @ 10.3%. Solution: Service Tax liability will be calculated as follows: Bill No. Amount received/ due 1,00,000 9,50,000 3,00,000 1,00,000 2,00,000 Date on which it was due Jun 30, 2011 Sep. 30,2011 Dec. 31, 2011 Jan 31, 2012 Mar 31, 2012 Cumulative amount since April 1, 2010 1,00,000 10,50,000 13,50,000 14,50,000 16,50,000 Tax liability

Registration

Exemption

100 101 102 103 104

No registration required Registration required Already registered Already registered Already registered

No Service Tax required Chargeable to Service Tax Chargeable to Service Tax Chargeable to Service Tax Chargeable to Service Tax

Nil 5,150 36050 46350 66950

IPCC _27e_Service Tax_Finance Act 2011 ____________________________ 68

No.1 for CA/CWA & MEC/CEC

MASTER MINDS

PART 6. GENERAL PROCEDURES


NO. 1. 2. 3. 4. 5. 6. 7. 8. 9. 10. 11. 12. 13. 14. QUESTION Explain the procedure for Registration of Service Tax. What is Service Tax Code number (STC)? Who is liable for Registration? What are the documents to be enclosed with, for Registration? Explain the circumstances that call for multiple registrations. What are the procedures to be followed in case of changes in existing certification of registration? Explain the procedure for surrender of Registration of Certificate. What are the due dates for the payment of Service Tax? What is the liability of a service provider, in case of excess collection of tax? Explain the Service Tax (Provisional Attachment of Property) Rules, 2008. An assessee has paid Service Tax in excess of his liability. Advise him, of the provisions in this regard. Write short notes on e-payment of Service Tax. Can Service Tax be paid in advance? What do you mean by e-filing of returns? Is there any facility of e-filing of Service Tax Returns? If yes, then which of the services are eligible for this facility? Write short notes on (a) person, (b) business entity, (c) taxable service, (d) service provider, (e) service receiver. What is the time limit for the issue of an invoice? What are the documents to be attached with, along with the return? Explain the procedure for filing Service Tax Returns. Explain the penal provisions relating to delayed furnishing of Service Tax Returns. [sec.70 and rule 7c]. Briefly explain Service Tax Return Preparers scheme. ABC A B A A B B B A C C C A B B C B B A A C

15. 16. 17. 18. 19. 20.

Single / Multiple Service Registration


Registration

Single Service

Multiple Service One premises Multiple premises Independent Accounting & Billing

One premises

Multiple premises

Centralized Accounting & Billing

Independent Accounting & Billing

Single Brand Name

Multiple Brand Name

Centralized Accounting & Billing

Single Registration

Centralized Registration

Independent Registration

Single Registration

Multiple Registration

Centralized Registration

Independent Registration

IPCC _27e_Service Tax_Finance Act 2011 ____________________________ 69

Ph:

0863 22 42 355

www.gntmasterminds.com

Q.NO.1. EXPLAIN THE PROCEDURE FOR REGISTRATION OF SERVICE TAX 1. Form: The application should be submitted in form No. ST-1 2. Time Limit: Within 30 days, from the date on which
Service tax is levied or whichever is later Business is commenced

3. Submission: The application should be submitted to the Superintendent of Central Excise or such other person, notified by the Central Government having his jurisdiction over the place of business of the service provider. 4. Documents to be enclosed: a. Proof of address. b. Copy of Permanent Account Number (PAN) card or PAN Allotment Letter. c. Articles of Association and Memorandum of Association (for companies). d. Copy of partnership deed (for partnership firms). e. In case of professionals, like CAs, CS, etc., who are members of professional institutes and have been granted a Certificate of Practice (COP), a copy of such certificate may also be attached. f. Extract of Board resolution, authorizing any of the directors / employees of the company to sign, deal and comply with Service Tax provisions.

g. Residential address of the proprietor/partners. h. Name and address of the Authorized Signatory. 5. Certificate of registration: a. The Jurisdictional Superintendent of Central Excise, after due verification of the application, will issue the certificate of registration, in form ST-2, within 7 days from the date of receipt of application. b. If the registration certificate is not granted within 7 days, the registration applied shall be deemed to have been granted. c. Where the application for registration, submitted by the assessee, contains more than one taxable service, the Certificate of Registration shall also indicate of all taxable services, provided by him. 6. Penalty: a. Rs.200 per day of default (or) Rs. 10,000, whichever is higher. (Rs.5,000 before April 8th, 2011) b. No penalty will be leviable if the assessee proves that there was reasonable cause for such a failure.

IPCC _27e_Service Tax_Finance Act 2011 ____________________________ 70

No.1 for CA/CWA & MEC/CEC


Q.NO.2. WHAT IS SERVICE TAX CODE NUMBER (STC)?

MASTER MINDS

1. STC is a 15 digit alpha-numeric code, obtained by the service provider, on an application made to the Jurisdictional Superintendent of the Central Excise. 2. It is a combination of: Particulars Permanent Account Number (PAN) Alpha-code (ST) Premises code Numeric Example: AABCC5588K-ST-001. 3. STC will be allotted within 3 working days, from the date of application, in the prescribed format, by the Assistant Commissioner / Deputy Commissioner. 4. It is mandatory to quote the STC number, on all documents relating to Service Tax. 5. In respect of e-filing of Service Tax Returns, STC is referred to as STP code, without which, e-filing is not possible. Q.NO.3. WHO IS LIABLE FOR REGISTRATION? Registration: Persons, who are liable to pay Service Tax, are required to register with the Central Excise Department. These persons of: 1. Service provider. 2. Specified Persons [Rule 2(1)(d)]: In the following cases, the specified person (and not the service provider) is liable to pay Service Tax. Category of Service No. of Digits 10 2 3

a. General Insurance Business. b. Insurance Auxiliary Service, provided Insurance Company by an Insurance Agent. c. Service provided by a Goods Person Liable: Specified Consignor/ Transport Agency, for transport of Consignee who pays or is liable to pay goods by road. freight, should pay the Service Tax.

Specified Person Insurer of Re-insurer

Specified consignor or consignee includes (i) Factory; (ii) Company; (iii) Corporation, (iv) Society; (v) Co-operative Society; (vi) Registered Dealer of Excisable Goods; (vii) Body Corporate;

d. Business Auxiliary Services, provided by Mutual Fund Distributors, in relation to distribution of Mutual Fund.

(viii) Partnership Firm. Service Tax should be paid by the Mutual Fund or the Asset Management Company, receiving such services.

IPCC _27e_Service Tax_Finance Act 2011 ____________________________ 71

Ph:

0863 22 42 355

www.gntmasterminds.com
by the Body receives the service [in of Services and received

e. Sponsorship Service provided to any Service Tax is to be paid Corporate or Firm, which Body Corporate or Firm. sponsorship service. f. Any taxable service provided by any Person receiving such person, from a country other than accordance with Taxation (provided from outside India India U/s 66A. in India)] Rules, 2006.

3. Persons notified by the Central Government Special Category Persons: a. Input Service Distributor: i. Office: Input Service distributor refers to an office of the
Manufacturer, or Producer of final products, or Provider of output service.

ii. Invoice: Which receives tax paid invoices (under Rule 4A of Service Tax Rules, 1994), towards the purchases of input services and issues invoice, bill or challan. iii. Purpose: For distributing the credit of Service Tax paid, on the said services, to such a manufacturer or producer or provider. b. Aggregate value of taxable services exceeds Rs.9 Lakhs:
Every person, liable to pay Service Tax shall apply for registration, in Form ST-1

within 30 days from the levy of tax. Normally, the applicant will be the service provider.
However, in exceptional cases, the service receiver with tax liability shall also

register himself.
Registration Certificate (RC) is issued in Form ST-2, within seven days. For all the

services provided by a person a single RC will do.


A separate RC is required for every place of business. However, the service

provider may opt for a single RC, if he follows centralized billing or centralized accounting, and wishes to remit the tax centrally. Q.NO.4. WHAT ARE THE DOCUMENTS TO BE ENCLOSED WITH, FOR REGISTRATION? An application for registration has to be enclosed with, along with the following documents: 1. Application in Form ST-1, in triplicate, duly signed. 2. Attested Copy of the PAN Card. 3. Proof of Address of the premises, which is required to be registered. 4. Copy of the document, governing the constitution of the organization (Partnership Deed in case of a partnership firm, Memorandum of Association in case of a company & Trust Deed in case of a trusts or associations). 5. Authority Letters, in favour of the person who is to collect the registration certificate, on the Letter head of the organization applying for the registration. 6. Power of Attorney in case the documents are signed by an authorized representative.

IPCC _27e_Service Tax_Finance Act 2011 ____________________________ 72

No.1 for CA/CWA & MEC/CEC

MASTER MINDS

Q.NO.5. EXPLAIN THE CIRCUMSTANCES THAT CALL FOR MULTIPLE REGISTRATIONS. 1. Multiple Proprietorship Firms, under different names: The applicant shall register individually, for each of such Proprietary Firm. 2. Multiple services: a. Commencement of services at the same time: The applicant, may mention, all the taxable services provided by him, in a single application. b. Commencement of services at different points of time: If the applicant has already registered for one service, but subsequently becomes liable for another category of service, then he should get his certificate endorsed for that category of service also. 3. Multiple Premises: a. Independent Billing and Accounting: Where taxable services are provided from more than one premises and, billing and accounting are done independently from each of such premises, separate application for registration is to be made in respect of each such premises or office. Certificate issuing authority shall be Jurisdictional Superintendent of Central Excise of the location where such premises are located. b. Centralised Billing and Accounting: Assessee can opt for registering only the premises or office from where such Centralised Billing is done or such Centralised Accounting Systems are located. Certificate issuing authority shall be Commissioner of Central Excise in whose jurisdiction the premises or offices from where centralised billing and accounting is done are located. The Commissioner of Central Excise granting Centralised Registration, shall send a copy of Form ST -2 (Centralised Registration) to all Commissioners in whose jurisdiction the Associated Premises are located. 4. Changes in Existing Certification of Registration: a. Change in information, provided in ST-1: Intimated in writing to the jurisdictional Assistant Commissioner within a period of 30 days of such a change. b. Change of place: A new Registration Certificate should be applied for, and the previous Registration Certificate should be cancelled. c. Transfer of business: Where the assessee transfers his business to another person, the transferee should obtain a fresh Certificate of Registration. Q.NO.6. WHAT ARE THE PROCEDURES TO BE FOLLOWED IN CASE OF CHANGES IN EXISTING CERTIFICATION OF REGISTRATION? 1. Intimation to Authorities: Additions or changes made to the information provided in Form ST -1 should be intimated in writing to the Jurisdictional Assistant Commissioner within a period of 30 days of such change in the following cases:

IPCC _27e_Service Tax_Finance Act 2011 ____________________________ 73

Ph:

0863 22 42 355

www.gntmasterminds.com

a. Change in line of taxable service, b. Change in Constitution of the Organisation, and c. Change in Place of business within the Jurisdictional Area. 2. Multiple Services: a. If an assessee providing multiple taxable services under single registration has opted to discontinue any one or more of such taxable services, then he should bring such cessation to the notice of the concerned Superintendent of Central Excise. b. The Superintendent of Central Excise will make an endorsement to that effect on the Certificate of Registration. Q.NO.7. EXPLAIN THE PROCEDURE FOR SURRENDER OF REGISTRATION OF CERTIFICATE. a. Circumstances:
Cessation of services, Change of place & Transfer of business.

b. Procedure for Surrender of Certification of Registration:


The assessee should surrender the Service Tax Registration Certificate (ST-2). Superintendent of Central Excise will cancel the Registration Certificate, after ensuring

that the assessee has paid all Service Tax dues to the Government, under the provisions of the Act. c. Non surrender of certificate:
There is no statutory requirement for the assessee to make an application for the

surrender of certificate.
Non-surrender of certificate does not attract any penalty. However, the assessee should furnish half yearly returns, even if no service is provided.

Q.NO.8. WHAT ARE THE DUE DATES FOR PAYMENT OF SERVICE TAX? 1. Service Tax is to be paid on a monthly / quarterly basis, depending upon the status of the service provider. Mode of Payment Status of Service Provider Last date for making Payment of Service Tax 6th day of the month immediately following the calendar month in which the service is deemed to have been provided as per POTR, 2011. However, for the month of March, Service Tax has to be deposited by 31st March.

Electronically through internet banking i.e. online Payment

Any person other than an individual or proprietary firm or partnership firm

IPCC _27e_Service Tax_Finance Act 2011 ____________________________ 74

No.1 for CA/CWA & MEC/CEC


Any person other than an individual or proprietary firm or partnership firm or partnership firm

MASTER MINDS
5th day of the month immediately following the calendar month in which the service is deemed to have been provided as per POTR, 2011. However, for the month of March, Service Tax has to be deposited by 31st March. 6th day of the month immediately following the quarter in which the service is deemed to have been provided as per POTR, 2011. However, for the month of March, Service Tax has to be deposited by 31st March. 5th day of the month immediately following the quarter in which the service is deemed to have been provided. However, for the month of March, Service Tax has to be deposited by 31st March.

Any mode other than internet banking

Electronically through internet banking i.e. online payment

An individual or proprietary firm or partnership firm

Any mode other internet banking

An individual or proprietary firm or partnership firm

2. Big assesses, with a Service Tax liability of Rs.10 lakhs and above, should pay tax electronically. For electronic payment, Service Tax Reg.No. is mandatory. 3. If the last date for paying tax is a public holiday, tax may be paid on the next working day. 4. Tax is paid by debit, to CENVAT account (if credit available) or through GAR 7 challan. 5. The date of presentation of cheque to the designated bank shall be deemed to be the date of payment of Service Tax (But the cheque should not be dishonoured). 6. Rounding off of tax: The payment of Service Tax should be round off, in multiple of rupees. Where such amount includes 50 paise or more, it should be increased to one rupee and if it is less than 50 paise it should be ignored. Q.NO.9. WHAT IS THE LIABILITY OF A SERVICE PROVIDER, IN CASE OF EXCESS COLLECTION OF TAX? Excess collection of tax Sec. 73A & 73B: 1. The service provider shall not collect tax in excess of what he pays to the Government. 2. In case his collection is more than his payment, he shall promptly deposit the excess with the Government or refund the amount collected in excess to the customer. 3. Any delay in this regard attracts payment of interest @ 18% p.a.

IPCC _27e_Service Tax_Finance Act 2011 ____________________________ 75

Ph:

0863 22 42 355

www.gntmasterminds.com

Q.NO.10. EXPLAIN THE SERVICE TAX (PROVISIONAL ATTACHMENT OF PROPERTY) RULES, 2008. These rules, introduced by Notification No.30/2008 ST dated 01.07.2008, with effect from 01.07.2008, provides as follows: a. The Assistant or the Deputy Commissioner of Central Excise, after due verification of the facts and circumstances of the case, may forward a proposal for provisional attachment of property belonging to a person, on whom a notice has been served under Section 73(1)/73A(3) of the Act, to the Commissioner, in the format prescribed in these Rules. b. The Commissioner may serve a show cause notice on such person who can make a submission, in this regard, within 15 days of service of the notice. c. Upon consideration of submission, the Commissioner may pass an order, to attach the property, provisionally. d. Value of property attached shall be of value, as nearly as, may be equivalent to that of the amount of pending revenue, against such person. e. The movable property of such person shall be attached, only if, the immovable property available for attachment is not sufficient to protect the interest of revenue. f. The said person or his representative shall not mortgage, lease, transfer, deliver or deal with the attached property, in any manner except with the previous approval of the Commissioner of Central Excise.

g. Every such provisional attachment shall cease to have its effect, after the expiry of a period of six months from the date of the service of the order passed. h. However, the Chief Commissioner of Central Excise may grant an extension, for a maximum period of two years. Q.NO.11.AN ASSESSEE HAS PAID SERVICE TAX IN EXCESS OF HIS LIABILITY. ADVISE HIM, OF THE PROVISIONS IN THIS REGARD.

a. Excess Service Tax Paid: W.e.f. 01.03.2007: If the assessee has paid any amount in excess of the amount required to be paid towards Service Tax liability for a month / quarter, then
a. the assessee may adjust such excess amount, paid by him against his Service Tax liability, for the succeeding month / quarter. b. the assessee cannot adjust the excess amount during the period after the succeeding month / quarter.

b. Adjustment against Excess Service Tax paid: An assessee can adjust his Service Tax liability against Service Tax paid previously, only if, he hasi. Paid excess Service Tax, ii. Refunded the amount collected (including Service Tax) to the client, for those services which are not wholly / partially rendered by him for any reason. iii. Filed the relevant details in respect of such suo moto adjustments done by him, at the time of filling Service Tax Returns, in Form ST-3.

IPCC _27e_Service Tax_Finance Act 2011 ____________________________ 76

No.1 for CA/CWA & MEC/CEC


c. Conditions for adjustment: i.

MASTER MINDS

Reasons for Excess Payment: Such excess amount paid is on account of reasons not involving interpretation of law, taxability, classification, valuation or applicability of any Exemption Notification.
General: Maximum amount, that can be adjusted for a relevant month / quarter, is Rs.2,00,000. For Centralised Registration Cases: No monetary limit, if excess amount is paid on account of delayed receipts of payments towards taxable services.

ii. Monetary Limit:

iii. Intimation to the Department: Details and reasons for such adjustment shall be intimated to the jurisdictional Superintendent of Central Excise, within 15 days from the date of adjustment. d. Renting of Immovable Property [Rule 6(C)]: i.
Applicability: Property owner paying Property Taxes, levied by the Local Authority.
Property Tax is paid, after the payment of Service Tax on the rental. Property Tax so paid, could not be deducted from rental, at the time of the payment of Service Tax (due to conditions in Notification 24 / 2007 dt. 22.5.2007, i.e. the Actual payment of Property Tax), and consequently excess Service Tax has been paid.

ii. Situation:

iii. Adjustment: Service Tax paid in excess, due to the payment of Property Tax after the payment of Service Tax can be adjusted in the following manner: Time Limit: Within 1 Year, from the date of payment of Property Tax. Monetary Limit: No limit has been set. Intimation: Details of adjustment should be intimated to the Jurisdictional Superintendent of Central Excise, within 15 days of such an adjustment.
Q.NO.12. WRITE SHORT NOTES ON E-PAYMENT OF SERVICE TAX. WHAT IS THE DUE DATE FOR PAYMENT, IN CASE OF E-PAYMENT OF SERVICE TAX? E Payment

Service Tax Liability Aggregate ST Liability Rs.10,00,000 In Previous Financial Year

Multiple Premises Aggregate ST Liability Rs.10,00,000 for Centralized Registration Aggregate ST Liability Rs.10,00,000 for Independent Registration

In the current financial year, e-payment of Service Tax is mandatory for all months.

Will not apply for each premises.

Will apply for each Independent Registration

IPCC _27e_Service Tax_Finance Act 2011 ____________________________ 77

Ph:

0863 22 42 355

www.gntmasterminds.com

a. Nature: Mandatory. b. Name: The System is called Electronic Accounting System in Central Excise and Service Tax (EASIEST). c. Applicability: Assessees, who have paid a Service Tax of Rs.10 lakhs or more during the preceding financial year. d. Due Date: Individuals / Proprietary Firm / Partnership Firm Other persons e. Qualifying Amount: i. Many Registered Premises: Rs. 10 lakhs would apply to each of the premises individually, if each such premises is separately registered for payment of Service Tax. 6th of the month, following the quarter of collection. In case of quarter ending in March, the tax should be paid on or before 31st March itself. 6th of the month, following the month of collection. In case of month ending in March, the tax should be paid on or before 31st March itself.

ii. Service Recipient paying Service Tax: The limit of Rs. 10 Lakhs includes Service Tax paid under Rule 2(1)(d), i.e. tax borne by the service recipients on taxable services, received by him. iii. Large Taxpayer Units: Cumulative Service Tax, paid by all registered premises of such large taxpayer, will be taken into account for the criterion of payment of Service Tax amount of Rs. 10 Lakhs. iv. Amount paid by CENVAT: Rs.10 lakhs limit includes total of Service Tax paid, i.e. by cash and by way of CENVAT Credit. v. Penalty: Upto Rs. 10,000. (Rs.5,000 before April 8th, 2011) Q. NO. 13. CAN SERVICE TAX BE PAID IN ADVANCE? IF SO, WHAT ARE THE INCIDENTAL CONDITION TO BE FULFILLED? (OR) HI-TECH Ltd. IS PROVIDING BUSINESS AUXILIARY SERVICES. IT HAS MADE AN ADVANCE PAYMENT OF SERVICE TAX, DURING THE MONTH OF JANUARY 2011 ON ITS OWN AND WANTS TO ADJUST THE SAME IN THE SUBSEQUENT PERIOD. THE COMPANY HAS APPOINTED YOU AS A CONSULTANT TO KNOW WHETHER IT CAN BE DONE SO. GIVE YOUR OPINION. a. Advance Tax: Every person, liable to pay Service Tax, can pay an amount as Service Tax in advance, to the credit of the Central Government. b. Adjustment on Accrual Basis: The assessee can adjust the advance Service Tax against the Service Tax liability, in the subsequent period, subject to the following conditionsi. Intimation: Assessee intimates the fact of such an advance payment of Service Tax to the Jurisdictional Superintendent of Central Excise within 15 days of the payment.

ii. Disclosure in Return: Indicate the details of advance payment and its adjustment (if any) in the subsequent return filed u/s 70. c. Nature of Payment: Optional.

IPCC _27e_Service Tax_Finance Act 2011 ____________________________ 78

No.1 for CA/CWA & MEC/CEC

MASTER MINDS

Q.NO.14. WHAT DO YOU MEAN BY e-FILING OF RETURNS? IS THERE ANY FACILITY OF e-FILING OF SERVICE TAX RETURNS? IF YES, THEN WHICH OF THE SERVICES ARE ELIGIBLE FOR THIS FACILITY? a. e-Filing is a facility for the electronic filing of Service Tax Returns by the assessee, through the Internet, by using a computer. b. The facility for e-filing of returns is ACES Automation of Central Excise and Service Tax. c. All services are eligible for e-filing of returns, subject to the general conditions listed. d. W.e.f. 1st October 2011 every Assessee shall subject the Half Yearly return electronically. Q.NO.15.WRITE SHORT NOTES ON (A) PERSON, (B) BUSINESS ENTITY, (C) TAXABLE SERVICE, (D) SERVICE PROVIDER & (E) SERVICE RECEIVER a. Person: Person includes both natural and artificial persons, i.e. Hindu Undivided Family (HUF), Firms, Companies, Corporations, Trusts, Institutions, Societies & Associations (incorporated or otherwise). b. Business Entity (w.e.f. 01.07.2010): Business Entity includes an Association of Persons, Body of Individuals, Company or Firm but does not include an individual. c. Taxable Service: Taxable Service means any service, provided or to be provided d. Service Provider:
Meaning: Service Provider is one who provides taxable service, as defined U/s 65(105)

and is generally liable to pay tax.


Exceptions: In the following cases, service receiver is liable to pay tax (i) General

Insurance, (ii) Services like Goods Transport Agency, (iii) Insurance Auxiliary Service, (iv) Sponsorship Services, (v) Asset Management Company or Mutual Fund Services etc. (vi) Import of Services
Constitution: In Service Tax Law, the constitution of the service provider can be (i)

Any Person, (ii) Commercial Concerns, (iii) Agency, (iv) Establishment, etc. e. Service receiver:
Meaning: Service receiver is the person, who receives or avails the service, provided

by the service provider. In some cases, the service receiver is liable to pay tax, and not the service provider.
Inclusions: Service receiver has not been defined under Service Tax Law and is

referred to as (i) Any person, (ii) Policyholder, (iii) Subscriber, (iv) Exhibitor, (v) Franchisee, etc.

IPCC _27e_Service Tax_Finance Act 2011 ____________________________ 79

Ph:

0863 22 42 355
Returns

www.gntmasterminds.com

Q.NO.16.WHAT IS THE TIME LIMIT FOR THE ISSUE OF AN INVOICE?

a. General:
Invoice / Bill should be issued within 14 days, from the date of -

Completion of the taxable service, (or) Receipt of any payment, towards the value of such a taxable service,

Whichever is earlier.

b. Provision of Continuous Service: Where a person provides continuous service for successive period of time, and the value of such service is determined or payable periodically, the service provider shall issue invoices, bills or challans within 14 days from the last day of the said period.
Example: Telephone Services. Note: Penalty for issuing incorrect invoice or not accounting invoices in books of account Rs.10,000 (Rs.5,000 before April 8th,2011) Q.NO.17. WHAT ARE THE DOCUMENTS TO BE ATTACHED ALONG WITH THE RETURN? The following documents are to be attached, along with ST-3: 1. Copies of GAR-7 challans, for the payments made. 2. Memorandum ST-3A, incase of provisional payment of tax. 3. In case of first return, the details of accounts maintained, in relation to Service Tax should be furnished to Superintendent of Central Excise, at the time of filing the first half yearly return i.e., ST-3, a list of all accounts maintained by him, in relation to Service Tax including the memorandum received from his branch office. Q.NO.18. EXPLAIN THE PROCEDURE FOR FILING SERVICE TAX RETURNS. Periodicity Nature Applicability Half yearly Mandatory. Return to be filed, even if Service Tax Collection is NIL or no service is rendered, in a particular period The persons liable to pay Service Tax to the Government are liable to file Service Tax Return. They are

a. Service provider, i.e. person providing taxable services. b. Specific persons under Rule 2(1)(d) of Service Tax Rules, 1994.
Due Date

a. 25th October for Half Year ending 30th September b. 25th April for Half Year ending 31st March.

IPCC _27e_Service Tax_Finance Act 2011 ____________________________ 80

No.1 for CA/CWA & MEC/CEC

MASTER MINDS

If due date is a Public Holiday then the next working day will be the due date. Form Many Services ST 3 (in Triplicate), indicating month/ quarter wise details for each of the taxable service rendered along with copies of relevant challans. Only one return for all the services. However, details of each taxable service should be shown separately Month wise details of:

Details to be disclosed

a. Amount received, towards the taxable value b. Amount received, in advance, towards taxable service to be provided c. Amount billed for exempted services and services exported, without the payment of tax d. Amount billed for services on which tax is to be paid e. Abatement claimed value f. Notification number of abatement and exemption g. Service Tax payable h. Education Cess payable i. GAR-7 challan, date and number j. Credit details for Service Tax provider/recipient a. GAR-7 Challans i.e. Proof of payment of Service Tax. b. Memorandum in form ST - 3A in case of provisional assessment.
At the time of filing the 1st Return every assessee should furnish a list of the following:

Enclosures

a. Records prepared / maintained for Provision of services (both taxable and not taxable) Receipt or procurement of input services First Return Payment for such input services Receipt, purchase, manufacture, storage, etc. of inputs and capital goods Records for other activities such as manufacture and sale of goods.

b. Other Financial records.


Nil Return Revised Return [Rule 7B] Even if no service has been provided during a half-year, and no Service Tax is payable; the assessee has to file a NIL return within the prescribed time limit. An assessee can file a revised return-

a. Time Limit: Within 90 days from the date of filing the original return. b. Reason: To rectify mistakes, if any, in the original return.

Q.NO.19. EXPLAIN THE PENAL PROVISIONS RELATING TO DELAYED FURNISHING OF SERVICE TAX RETURNS. [SEC.70 AND RULE 7C]

IPCC _27e_Service Tax_Finance Act 2011 ____________________________ 81

Ph:

0863 22 42 355

www.gntmasterminds.com
Amount of Penalty Rs. 500 Rs. 1,000 Rs. 1000 + Rs. 100 for every day from 31st day, Or Rs. 20,000 whichever is LOWER. (Rs.2,000 before April 8th, 2011)

The penalty for delayed filing of returns is based on the period of delay Period of delay from the prescribed date for furnishing the returns 15 days Beyond 15 days and within 30 days

Beyond 30 days

Note: CEO empowered to reduce or waive penalty (late fees) for delay in filing return where gross amount of Service Tax is NIL. Q.NO.20. BRIEFLY EXPLAIN SERVICE TAX RETURN PREPARERS SCHEME? a. Definitions:
Service Tax Return Preparer: It refers to an individual, who has been authorized to act

as a Service Tax Return Preparer, under a scheme framed under this section.
Specified Classes of persons: It refers to persons specified in the scheme, who are

required to furnish a return, required to be filed under Section 70. b. Power of Board: The CBEC is empowered to frame a scheme, for furnishing return of income, by any specified classes, through a Service Tax Return Preparer. c. Duty of Service Tax Return Preparer: Every Service Tax Return Preparer should assist the specified classes of persons, to prepare and furnish the Service Tax Return, in the manner specified in the scheme. d. Structure of the Scheme: The scheme, framed by the Board, may provide for the following
Manner, in which and the period for which the Service Tax Return Preparer shall be

authorized,
Educational and other qualifications to be possessed, and the training and other

conditions required to be fulfilled, by a person, to act as a Service Tax Return Preparer,


Code of Conduct, for the Service Tax Return Preparer, Duties and Obligations of the Service Tax Return Preparer, Circumstances, under which the authorization given to a Service Tax Return Preparer

may be withdrawn,
Any other matter, which is required to be specified by the Scheme for the purposes of

this section

IPCC _27e_Service Tax_Finance Act 2011 ____________________________ 82

No.1 for CA/CWA & MEC/CEC Illustrations

MASTER MINDS

1. Determination of Service Tax Liability: J.C Professionals, a partnership firm, gives the following particulars relating to the services provided to various clients by them, for the half year ended on 30.09.2011: a. Total bills raised are for Rs. 8,75,000, out of which a bill of Rs. 75,000 was raised on an approved International Organization, and payments of bills for Rs.1,00,000 were not received till 30.09.11. b. An amount of Rs.50,000 was received as an advance from XYZ Ltd., on 25.09.11 to whom the services were to be provided in October, 2011. You are required to work out the (a) Taxable value of services (b) Amount of Service Tax payable. Solution: Computation of Taxable Value of Service and Tax Payable thereon Particulars Total value of bills raised Less: Amount raised on approved International Organization [exempt] Add: Amount of advance received from XYZ Ltd. (See Note) Total Taxable Value Tax Payable on above Service Tax @ 10% Add: Education Cess @ 2% Add: SHEC @ 1% Total Tax Payable 85,000 1,700 850 87,550 Rs. 75,000 75,000 8,00,000 50,000 8,50,000 Rs. 8,75,000

Note : It is assumed that invoice is raised within 14 days from the date of receipt of the amount i.e. before 8th October, 2011. 2. Mr. Bharat is a registered service provider. He transfers his business to Mr. Rakesh on 31st July, 2011. Explain the requirement to be complied with, by Mr. Bharat and Mr. Rakesh on such a transfer under the provisions of Service Tax. Solution: a. Bharats Duty: Where the assessee transfers his business to another person, he should surrender the Certificate (if he does not provide any other taxable service). In the given case, Mr. Bharat should surrender the Certificate of Registration. b. Rakeshs Duty: Mr. Rakesh, to whom the business is being transferred, has to apply for registration. In the given case, the business is transferred on 31st July, 2011. Assuming that Mr. Rakesh commences the newly acquired business on the same date, he should apply for registration before 30th August, 2011.

IPCC _27e_Service Tax_Finance Act 2011 ____________________________ 83

Ph:

0863 22 42 355

www.gntmasterminds.com

3. Y & Co. seeks your advise, for the following, in the context of Service Tax: It wants to file revised Service Tax Return even though the original return was filed belatedly. Solution: Filing of revised Service Tax Return: Service Tax Return can be revised within 90 days, from the date of filing of the original return. This rule is applicable, even if the original return was submitted belatedly. 4. Suyogya Consultancy Services (SCS) is engaged in providing Management Consultancy Service during the financial year 2011-12. Examine, whether SCS shall be liable to pay the late fee for delay in furnishing the return for the half yearly period ending 30th September 2011 in the following cases: a. It files its return of Service Tax on 26th October, 2011 (25th October was declared as a public holiday) b. It files its return on 15th November 2011. Solution: Due Date for half yearly period ending 30th September, 2011 is 25th October, 2011. Date of filing return
th

Reason Since October 25 is a public holiday, the assessee can file the return on the next immediately succeeding working day i.e., October 26th, 2011 Period of delay is 21 days (26th Oct to 15th Nov 2011)

Late Fee No Late Fee Rs.1,000

Oct 26, 2011 Nov 15, 2011

5. Peter is running a sound recording studio and has obtained registration for the purpose of payment of Service Tax. He proposes to transfer his entire business to Akash. It is necessary for Akash to obtain a fresh Certificate of registration. Due reason to your answer. Solution:

a. Change in information provided in ST-1. Additions are changes made to the information provided in form ST-1 should be intimated in writing to the Jurisdictional Assistant Commissioner with in a period of 30 days of such change. b. Multiple Services:
If an assessee providing multiple taxable services under single registration has opted to discontinue any one or more of such taxable services, then he should bring such cessation to the notice of the concerned Superintendent of Central Excise. The Superintendent of Central Excise will make an endorsement to that effect on the Certificate of Registration.

IPCC _27e_Service Tax_Finance Act 2011 ____________________________ 84

No.1 for CA/CWA & MEC/CEC Self Test Questions

MASTER MINDS

1. Discuss, whether the following persons are liable for registration, under Service Tax. If yes, when is the registration required? a. A provider of taxable services, whose aggregate value of taxable services is Rs. 8,00,000 up to December 31st, 2011. b. An input service distributor who starts his business on January 1st, 2012. c. A provider of taxable services, under a brand, which is not yet registered, provided services of Rs.8,00,000, up to January 31st, 2012. 2. Briefly explain the provisions in the Service Tax Rules, 1994 relating to furnishing of list of records at the time of filing of records and at the time of filing of return for the first time. 3. Briefly explain the following, with references to the Service Tax Rules, 1994, and the Finance Act, 1994:
a. Gross amount charged b. e-payment of Service Tax.

4. State briefly, whether the persons are liable to apply for registration, under the Finance Act, 1994, and Service Tax (Registration of Special Category of Persons) Rules, 2005, and if so, from which date: a. An input service distributor, who starts his business, with effect from 1st January, 2011 b. A provider of taxable service, under an unregistered brand name of another person.
Aggregate value of taxable services was Rs. 6,00,000 upto 31.3.2011.

5. When could an assessee surrender his Certificate of Registration under Service Tax Law? 6. What is the due date for monthly / quarterly payment of Service Tax? 7. What do you understand by Centralised Registration? 8. Answer the following, with reference to the Finance Act, 1994, and the Rules made there under, relating to Service Tax: Intimation regarding change in details, furnished by an assessee, in Form ST-1. 9. Calculate the value of taxable service of X Transport Company, engaged in the business of transport of goods by road. Give reasons for taxability or exemption of each item. No freight is received from any of the specified category of Consignor/Consignee. Suitable assumptions may be made, wherever required. X does not avail CENVAT credit:
a. Total freight charges received by X, during the year b. Freight charges received for transporting fruits Rs.13,50,000 Rs.1,25,000

c. Freight collected for transporting small consignment for persons, who paid less than Rs. 750 for each such consignment Rs. 75,000 d. Freight collected for transporting goods in small vehicles for persons, who paid less than Rs. 1,500 per trip. Rs.1,50,000

IPCC _27e_Service Tax_Finance Act 2011 ____________________________ 85

Ph:

0863 22 42 355

www.gntmasterminds.com

10. With references to the Finance Act, 1994, as amended and the rules made there under, relating to Service Tax, state whether registration is required or not, in the case of the following persons or class of persons: a. Input service distributor. b. Small service provider whose aggregate value of taxable service is Rs. 3,50,000 per annum. c. Indian based recipient of taxable services, provided from abroad, by a non-resident not having any place of business in India. 11. An assessee, who has collected Service Tax from a client, is unable to perform the service. Briefly explain the situations in which and the conditions subject to which, he can adjust the Service Tax relating to above, against his forthcoming Service Tax liability.
Hint: Adjustment against excess Service Tax paid.

Objective Type Questions


1. Service Tax extends to the a) Whole of India b) Whole of India except J & K c) Whole of India including J & K 2. Levy of Service Tax is on a) Receipt basis b) Rendering basis c) Rendering & receipt 3. In case of Service Tax, reliance is on collection through a) Voluntary compliance b) Service receiver c) Service provider 4. The act for levy of Service Tax is a) Service Tax act, 1994 b) Finance Act, 1994 c) Circulars & Notifications 5. In India, the approach to Service Tax is a) Comprehensive coverage b) Selective coverage c) Partly selective & Comprehensive 6. Services rendered to an Export Oriented Unit a) Taxable b) Not Taxable c) Based on the service 7. The Certificate of Registration is in Form a) ST 1 b) ST 2 c) ST 3

8. If Certificate of Registration is not received within __ days, it is deemed as registered. a) 15 b) 7 c) 30 9. Under Service Tax Law, all records should be preserved for a) Atleast 5 years b) 5 years c) More than 5 years 10. When service is provided on 4th August and advance payment received on 29th July 2010, cut off date for issue of Invoice is __ August b) 12th c) 18th a) 11th 11. In case of public authorities performing statutory functions, a) Services provided by them are exempt b) Services provided to them are exempt c) Services provided for them are exempt 12. Gross amount charged includes the payment by a) Cheque b) Credit card c) Both 13. Services provided during warranty period is: a) Exempt b) Not taxable c) Taxable

IPCC _27e_Service Tax_Finance Act 2011 ____________________________ 86

No.1 for CA/CWA & MEC/CEC


14. When the due date for payment of Service Tax is a public holiday, payment can be made on a) Next working day b) Preceding working day c) Within 3 days of grace from the due date 15. The amount of Service Tax shall be rounded off to the nearest a) one b) Ten d) Hundred 16. If Services are not provided during a period, Service Tax return. a) NIL return should be filled b) Need not be filed c) NIL return need not be filed 17. The threshold limit for exemption under Service Tax is Rs. a) 10 lakhs b) 9 lakhs c) 8 lakhs 18. Income Tax deducted at source is on the a) Value of service b) Value of service Service Tax amount c) Value of Service + Service Tax amount 19. Services provided by Central or State Government are taxable unless; a) These are statutory services b) Consideration is received c) Consideration is received in nonmonetary form 20. I-store files the return for the period ending 31st March 2010 on 20th April 2010. What is the time limit for revision of the return filed? a) 19th July 10 b) Revision not possible c) 24th July 10 21. The interest payment of Service Tax is _ a) 13% p.a. b) 12% p.a. c) 13% per month

MASTER MINDS
22. Mr. Xylam, is registered for the following services CHA, C&F Agent, Business Auxiliary Services & Cargo Handling Services, from various premises at T.Nagar, Trichy, Tambaram & Tuticorin, respectively. The main branch at T.Nagar accounts for all the transactions. What is the nature of registration. a) Centralised Registration b) Independent Registration c) Multiple Registration 23. Service is rendered on 10th September, 2010, advance is received on 18th August, 2010. Whereas the service was brought into the taxable net from 1st September, 2010. Determine the taxability. a) Taxable b) Not taxable c) Exempted 24. In case of NIL Service Tax, during a specified period, penalty for delayed filing of Service Tax Returnsa) Cannot be waived b) Can be waived by CEO c) Waiver is not applicable 25. Service Tax provisions are applicable to a) Exclusive economic Zone b) Export Oriented Unit c) Whole of India. not

Answers for Objective Type Questions


1. 6. 11. 16. 21. B B A A A 2. 7. 12. 17. 22. A A A A A 3. 8. 13. 18. 23. A B C B A 4. 9. 14. 19. 24. B B C C B 5. 10. 15. 20. 25. B A A A C

IPCC _27e_Service Tax_Finance Act 2011 ____________________________ 87

Ph:

0863 22 42 355
QUESTION Write short notes on self assessment.

www.gntmasterminds.com
PART 7. ASSESSMENT
ABC B A A B C C B B

NO. 1. 2. 3. 4. 5. 6. 7. 8.

What are the conditions for provisional assessment? Discuss the provisions as to show cause notice (sec.73). Write short notes on deposit of excess duty / tax collected. Write short notes on interest on amounts collected in excess of duty (sec. 73B). Write short notes on the payment of the interest on duty demanded. Write short notes on penalty for the failure to pay tax (sec.76). Write short notes on the levy of penalty for suppression.

Q.No.1. WRITE SHORT NOTES ON SELF ASSESSMENT. 1. Applicability: All Assessees. 2. Assessment: Assessee should assess the tax due, on the services provided by him. 3. Form: The half-yearly return, in Form ST-3, is both a return and a self assessment form. 4. Submission: The return should be furnished to the Superintendent of the Central Excise in the prescribed manner (i.e. in Form ST 3, before 25th April or 25th October, as the case may be). Q.No.2. WHAT ARE THE CONDITIONS FOR PROVISIONAL ASSESSMENT? 1. Circumstance for Provisional Assessment: Provisional Assessment is made, when a. Situation: The assessee is not in a position to correctly calculate the tax payable for the month or quarter b. Time: on the date of deposit of tax. 2. Condition: a. Request to AC / DC: The assessee should make a request to the jurisdictional AC / DC, for making provisional assessment. b. Reasons: Request should be supported by reasons, for payment of Service Tax, on provisional basis. c. Granting of Permission: AC / DC, on receipt of such request, may allow payment of Service Tax on provisional basis. He shall specify the value to be considered, for provisional assessment. 3. Filing of Memorandum: a. Memorandum: The assessee, who has opted for provisional assessment, has to file a memorandum in Form ST-3A along with the half yearly return in Form ST-3.

IPCC _27e_Service Tax_Finance Act 2011 ____________________________ 88

No.1 for CA/CWA & MEC/CEC

MASTER MINDS

b. Contents of Memorandum: ST-3A provides the details of the difference between the Service Tax deposited and Service Tax liable to be paid for each month or quarter. c. Calling for Documents: AC / DC can call for documents etc., to verify the details of Form ST-3A. d. Order of Assessment: AC / DC shall pass the final assessment order, within 6 months from the date of communication of provisional assessment. 4. Tax Payable / Refundable on Final Assessment: a. Order of Assessment: The Final Assessment Order may Require the assessee to pay Service Tax in addition or Entitle him to refund of excess tax paid, if any, any

b. Interest on Tax Payable:


Situation: Additional Service Tax payable on final payment. Rate of Interest: @18% p.a Amount on which payable: Additional Service Tax payable. Period of Interest: From the first day of the month, succeeding the month for which

such amount is determined, till the date of payment there of. c. Interest on Tax Refundable:
Situation: Assessee is entitled for refund, as per final assessment order. Rate of Interest: @ 6% p.a. Amount on which interest determined: Amount of refund due. Period of Interest: From the first day of the month, succeeding the month for which

refund is determined, till the date of refund. Q.NO.3. DISCUSS THE PROVISIONS AS TO SHOW CAUSE NOTICE (SEC.73). 1. Where any Service Tax has not been levied or paid, or has been short levied or short paid or erroneously refunded, then the Central Excise Officer can issue a show cause notice, requiring, as to why the person should not pay the amounts specified in the notice. 2. The show cause notice can be issued, even if the non-levy etc. was on the basis of any approval, acceptance or assessment, relating to rate of the Tax or valuation. 3. The show cause notice has to be issued within one year from the relevant date. 4. Where the tax has not been levied or paid or short levied or short paid or erroneously refunded by reason of fraud, collusion or any wilful misstatement, suppression of facts, or contravention of any of the provisions of the Act or Rules made there under, with an intention to evade payment of tax, the show cause notice can be issued, within five years from the relevant date. 5. Where the service of the show cause notice is stayed by an order of a court, the, period of stay shall be excluded, while computing the limitation period. 6. The officer, after considering the representation, if any, made by the person on whom the notice is served, shall determine the amount of tax, which shall not be in excess of the amount specified in the notice.

IPCC _27e_Service Tax_Finance Act 2011 ____________________________ 89

Ph:

0863 22 42 355

www.gntmasterminds.com

7. Where any notice has been served by the Central Excise Officer, a. If the demand is on account of fraud, suppression, collusion, willful misstatement or contravention of the act or rules with an intention to evade the payment of tax, etc., where it is possible to do so, the officer shall determine the amount, within a period of one year from the date of service of notice. b. In any other case, where it is possible to do so, officer shall determine the amount within a period of six months, from the date of service of notice. Q.NO.4. WRITE SHORT NOTES ON DEPOSIT OF EXCESS TAX / TAX COLLECTED. Sec. 73A Excess tax collected from the buyer, to be deposited with the Government: 1. Every person, who is liable to pay tax under the Act or Rules, and has collected any amount in excess of the tax assessed or determined and paid on any taxable services under the Act or Rules from the buyer, in any manner as representing tax on service, shall forthwith pay the amount collected, to the credit of the Central Government. 2. Where any amount so required is to be paid, has not been paid, the Central Excise Officer may serve a show cause notice on the person liable, to pay such amount. 3. The Officer, after considering the representation, if any, made by the person, on whom the notice is served, shall determine the amount due from, and thereupon such person shall pay the amount so determined. 4. The amount paid to the Government, under this provision, shall be adjusted against the tax payable by the person on finalisation of assessment or any other proceeding for the determination of tax. Where, after such adjustment there is any surplus, the amount shall be credited to the Consumer Welfare Fund or refunded to the person, who has borne the incidence of tax, in accordance with the provisions of Sec. 11B. Such a person is required to make an application, within 6 months from the date of public notice, to be issued by the Assistant Commissioner of Central Excise, for the refund of such surplus amount. Q.NO.5. WRITE SHORT NOTES ON INTEREST ON AMOUNTS COLLECTED IN EXCESS OF TAX (SEC. 73B).

1. Where any amount is collected in excess of tax, the person, who is liable to pay such an amount in terms of Sec.11D, shall also be liable to pay interest at such a rate not less than 10% as may be notified by the Government in the Official Gazette (Government has notified 18% w.e.f 1st April 2011). 2. Where the amount becomes payable consequent to an order passed by the Board U/s 37B and the assessee pays the amount in full, without reserving any right of appeal, within 45 days from the date of order, no interest is payable. 3. Where the amount determined, U/s 11D, is increased or decreased by the Appellate Authorities, the interest shall vary accordingly.

IPCC _27e_Service Tax_Finance Act 2011 ____________________________ 90

No.1 for CA/CWA & MEC/CEC

MASTER MINDS

Q.NO.6. WRITE SHORT NOTES ON PAYMENT OF INTEREST ON TAX DEMANDED. Interest (Section 75): Where a person liable to pay tax fails to credit the tax or any part thereof to the account of the Central Government, he shall, pay by simple interest, at such rate not less than 10% as may be fixed by the Central Government by notification in the Official Gazette. Interest is calculated for the period, by which crediting of the tax or any part thereof, is delayed. As of now Government has notified 18% (w.e.f. 1st April 2011) as the applicable rate of interest. Note: W.e.f. 8th April, 2011 the rate of interest U/s.73B & U/s.75 for delay in payment of tax to be reduced by 3% in case the value of taxable services provided by service provider during the preceding financial year or during any of the financial years covered by the Show Cause Notice, as the case may be, does not exceed Rs.60 Lacs. Q.NO.7. WRITE SHORT NOTES ON PENALTY FOR THE FAILURE TO PAY TAX (SEC.76). Where any person liable to pay Service Tax, fails to pay such a tax, he shall, in addition to such tax and interest under Section 75, pay a penalty a. Rs.100 per day during which the failure continuous (Rs. 200 before April 8th, 2011) b. 1% per month during which the failure continuous (2% per month before April 8th, 2011), which ever is higher. However, penalty cannot exceed 50% of Service Tax payable (it is 100%, before April 8th, 2011) c. If penalty is levied U/s 78, no penalty can be levied U/s 76. Example: X fails to pay Service Tax of Rs. 10,00,000 by the 5th of March. The amount is paid on 15th of March. Default is 10 days and the penalty is calculated as under: 1 x Rs.10,00,000 x 10/31 = Rs. 3,226 (2 x Rs.10,00,000 x 10/31 = Rs.6,452 before April 8th) or Rs. 100 per day for 10 days i.e. Rs.1,000 (Rs.2,000 before April 8th, 2011) whichever is higher i.e., penalty Rs.3,226. Q.NO.8. WRITE SHORT NOTES ON LEVY OF PENALTY FOR SUPPRESSION. General Penalty (Section 77): 1. Failure to take Service Tax registration. 2. Failure to furnish information called by CEO, produce document called by CEO or appear before CEO in response to Summons or produce documents in an inquiry. Penalty (for 1, 2 points): At discretion of CEO upto Rs.10,000/- or Rs.200/- for every day during which such failure continues, whichever is higher. 3. Failure to maintain / retain books of accounts & other documents. 4. Failure to pay tax electronically through internet banking when mandatory. 5. Incorrect and incomplete details on the invoice or failure to account invoice in the books of accounts. 6. Any contravention of the provisions or rules for which no separate penalty is provided.

IPCC _27e_Service Tax_Finance Act 2011 ____________________________ 91

Ph:

0863 22 42 355

www.gntmasterminds.com

Penalty (for 3, 4, 5, 6 points): Upto Rs.10,000/- for each default, maximum limit Rs.10,000/-. Penalty for suppression etc. (Section 78): 1. Where Service Tax has not been levied or paid or is short levied or short paid or has been erroneously refunded by reason of fraud, collusion, willful misstatement, suppression of fact or contravention of the provisions of the Act or Rules, with an intention to evade the payment of tax, then the person shall also be liable to pay a penalty, which shall not be less than the tax amount. 2. Where true & completed details of the transactions are available in the specified records, penalty shall be reduced to 50% of Service Tax. Such penalty is restricted to 25% of Service Tax, if the amount of Service Tax, interest and such penalty is paid within 30 days of the date of communication of the adjudicating order. In case the value of taxable services provided by service provider during the preceding financial year or during any of the financial years covered by the Show Cause Notice, as the case may be, does not exceed Rs.60 Lacs, the period of 30 days for such payment is extended to 90 days. No penalty in certain cases (Section 80): No penalty can be imposed under Section 76 or Section77 or Section 78, where the assessee proves that there was a reasonable cause for the failure.

Illustrations
1. Mr. Saravanan has collected a sum of Rs.15,000, as Service Tax from a client mistakenly, even though no Service Tax is chargeable for such a service. Should the amount so collected be remitted to the credit of the Central Government? Solution: a. Excess Collection [Sec.73A]: The assessee who has collected an amount in excess of the amount required to be collected, under Service Tax Law, should deposit such an amount with the Government. If not paid, the Central Excise Officer (CEO) shall serve a show cause notice. b. Interest on Amount collected in Excess [Sec.73B]: Where excess amount has been collected from the recipient of services, the person liable shall pay the excess amount together, with interest at the rate of not less than 10% and not exceeding 24%. It shall be payable from the 1st day of the month succeeding the month in which amount should have been paid till the date of actual payment. c. Conclusion: Mr. Saravanan should deposit Rs.15,000 collected as Service Tax with the Government which can be adjusted against Service Tax Liability at a later point of time. 2. X Ltd. has obtained a CENVAT credit of Rs. 55,000 by fraud. The fraud is detected by the Central Excise Officer on March 13th, 2011. A show cause notice is issued to X Ltd. on March 15th, 2011. The Central Excise Officer (after consideration represented by X Ltd.) Determine the quantum of tax payable on May 16th, 2011: Tax payable (i.e., CENVAT credit obtained by fraud) Interest Penalty (at the rate of 200% of Rs. 55,000) Amount payable The above order is communicated to X Ltd. on May 20 , 2011.
th

55,000 7,200 1,10,000 1,72,200

IPCC _27e_Service Tax_Finance Act 2011 ____________________________ 92

No.1 for CA/CWA & MEC/CEC

MASTER MINDS

Solution: If X Ltd. pays the following amount on or before June 19th, 2011 (i.e., 30 days from May 20th, 2011), the quantum of penalty will be reduced to 25% of Rs. 55,000 (i.e., Rs. 13,750) Tax payable (i.e., CENVAT credit obtained by fraud) Interest Penalty (at the rate of 25% of Rs. 55,000) Amount paid on or before June 19 , 2011
th

55,000 7,200 13,750 75,950

If the payment is not made up to June 19th, 2011 (or the amount paid up to June 19th, 2011 is lower than Rs.75,950), the quantum of penalty will not be reduced to 25%. In such a case, the penalty will be Rs.1,10,000, as determined by the concerned officer.

THE END

IPCC _27e_Service Tax_Finance Act 2011 ____________________________ 93

Das könnte Ihnen auch gefallen